You are on page 1of 28

1.

A 42 years old male comes to the Siloam hospital


with muscular weakness, mild headache after
eating salted fish. Usually he drinks only small
amount of water. He said that usually his blood
pressure is 100/70 mmHg. His blood pressure
140/90 mmHg. The doctor sends him to the
laboratory. The result shows serum sodium is 150
mEq/L (N: 135 - 155), potassium is 3.5 mEq/L (N:
3.5 5.0), random blood glucose 150 mg/dl (N
<140).
In this condition, what will the kidney do?
A. Increase glomerular filtration rate
B. Increase obligatory water reabsorption
C. Increase facultative water reabsortion
D. Increase renin secretion
E. Increase aldosteron secretion
2. To cover the above function, what mechanism is
used?
A. Increase cardiac output
B. Increase frequency of the heart
C. Increase secretion of renin
D. Increase secretion of antidiuretic
hormone
E. Increase secretion of aldosteron
3. Which factor support the above mechanism?
A. Filtration rate of the glomeruli
B. Active transport of the tubular cell
C. High osmolality of the interstitium of
medulla
D. Obligatory water reabsortion
E. Osmolality of the filtrate in distal
convoluted tubule
4. If the urinalysis is also done, what will you see
about the glucose in urine?
A. positive ++++
B. positive +++
C. positive ++
D. positive+
E. negative
5. In a kidney laboratory, using a micropipette, we
can measure the hemodynamic pressure of
afferent arteriole, pressure in Bowman capsule,
and osmotic pressure in glomerular lumen. If the
pressure in afferent arteriole is 40 mmHg, in
Bowman capsule 1 mmHg, the osmotic pressure
of glomerulus is 30 mmHg, the effective filtration
pressure in glomerulus is: (40-1-30)
A. 71 mmHg
B. 69 mmHg
C. 11 mmHg
D. 9 mmHg
E. 9 mmHg
6. A 63-year-old-man, with a history of heart failure
and edema fails to respond adequately to
maximum
recommended
dosages
of
chlorthalidone.

Which of the following is the most likely


appropriate for restoring the diuretic response?
A. add with hydrochlorothiazide
B. add with metolazone
C. replace with furosemide
D. replace with hydrochlorothiazide
E. increasing the dose of chlorthalidone
7. A 75-year-old woman with hypertension is being
treated with thiazide. Her blood pressure is
decrease to 120/76 mm Hg. After several months
on the medication, she complains of being tired
and weak. Which of the following substance in
analysis of blood indicates low values?
A. calcium
B. uric acid
C. potassium
D. sodium
E. glucose
8. A jaundiced 1-week premature infant with elevated
free bilirubin is seen in the premature baby
nursery. The mother had received antibiotic
combination for a urinary tract infection (UTI) 1
week before delivery.
Which of the following is the most likely cause of
the babys kernicterus?
A. cefixime
B. amoxicillin
C. azithromycin
D. erythromycin
E. cotrimoxazole
9. A 4-year-old-boy was brought to hospital with
puffiness around the eyes, especially in the
morning and swelling over the legs. His mother
also complaint he has foamy urine. Urine sample
2
shows protein ++++ (>3.5 per 1.73 m per 24
hour), albumin level 1.5g/dL (normal=3.5-5g/dL).
What is the principal management in this patient?
A. Albumin
B. Furosemide
C. Prednisone
D. Thiazide
E. Cyclosporin
10. A 13 year-old-girl was brought to emergency unit
with seizure. Her mother complaint she had fever 2
weeks ago, but has recovered. Now she has
swelling around the eyes and legs and she has
dark urine. From physical exam, she is
unconscious. Her blood pressure is 180/120
mmHg. Urine sample shows erythrocytes 20/hpf
and protein +. Albumin level within normal limit.
What other laboratory findings to confirm the
diagnosis in this patient?
A. Decreased anti streptolisin O (ASO),
increased DNase-B, decreased C3
B. Increased anti streptolisin O (ASO),
decreased DNase-B, decreased C3
C. Decreased anti streptolisin O (ASO),
increased DNase-B, increased C3

D. Increased anti streptolisin O (ASO),


decreased DNase-B, increased C3
E. Increased anti streptolisin O (ASO)
(untuk faringitis), increased DNaseB(pd nefritik/strepto), decreased C3
11. A 70-year-old male with tuberculosis, he was
treated with daily streptomycin for 1 week. After
the last injection, his urine output diminished and
the laboratory test revealed ureum 80 (20 40)
mg/dL, creatinine 3.0 (0.5 - 1.5) mg/dL. (AIN ada
proteinuria, ATN ga daa proteinuria)
What is the most likely diagnosis in this patient?
A.
B.
C.
D.
E.

Urinary tract infection


Sepsis
Glomerulonephritis
Chronic renal failure
Acute tubular nephritis

12. What is the most possible cause in this patient?


(AIN hypersentivity, ATN ischemic ata toxin)
A. Renal ischemia
B. Direct toxic injury
C. Complex immune response
D. Urinary obstruction
E. Drug hypersensitivity
13. A 3 year old boy was brought to emergency unit
with severe diarrhea since three days ago. On
arrival he is not fully alert. His vital sign are as
follows: blood pressure is 80/60 mmHg, pulse is
not palpable, respiratory rate was 36 breaths per
o
minute, and body temperature was 38.4 C. On
physical exam there is sunken anterior fontanel,
dry mucous membranes, sunken eyes, lack of
tears, poor skin turgor, and the capillary refill more
then 2 seconds. The laboratory test in this patient
reveal haemoglobin 13 g/dL(11.5-15.5 g/dL),
3
hematocrit 46% (35-45%), WBC 7,000/mm (5,5003
3
15,500/mm ),
platelets
count
260,000/mm
3
(150,000-400,000 mm ), urea nitrogen 40 mg/dL
(5-18 mg/dL), and creatinine 1.7 mg/dL (0.3-0.7
mg/dL). Urine specific gravity is 1.040 (N: 1.003
1.030)
What is the most likely diagnosis in this patient?
A. Acute tubular nephritis
B. Prerenal acute kidney injury
C. Intrinsic acute kidney injury
D. Postrenal acute kidney injury
E. Chonic renal failure
14. A 10 year old girl was brought to your clinic with
dysuria since 5 days ago. On physical exam she is
healthy with no significant findings. From urine
culture is found Eschericia coli wih CFU 100,000.
What is the most likely diagnosis in this patient?
A.
B.
C.
D.
E.

Acute pyelonephritic
Reflux nephropathy
Uncomplicated urinary tract infection
Complicated urinary tract infection
Chronic urinary tract infection

A 55-year-old man refers to ICU after the procedure


open reduction of femur fracture and bleeding intra
operative. At the third day he became apatis, fever
38,5 C with blood pressure 110/60 mmHg, pulse 55
x/min, respiratory 20x/min, and urine output decreased
to 10 cc/hour in 24 hours. At laboratory examination
findings creatinine 4 mg/dL (N: 0.5-1.5), ureum 100
mg/dL (N: 20-40), sodium 130 mEq/L (N: 135-155),
potassium serum 7.5 mEq/L (N: 3.5-5.5), blood sugar
200 mg/dL (N: <140).
From blood gas analysis finding pH 7.1, pCO2 30
mmHg, pO2 90 mmHg,
HCO3 18 mEq/L. ECG examination finds widening of
QRS wave.
15. What is the most critical condition can cause life
threatening the patient?
A. nothing for awareness
B. ureum and creatinine at very high lvl
C. hyperkalemia (CKD, AKI)
D. metabolic acidosis
E. severe hyponatremia
16. What is the treatment for the critical condition
above?
A. Waiting for the general condition of the
patient became more better
B. Give diuretic until the urine output will
be normally
C. Hemodialysis
immediately
(hiperkalemi parah)
D. Give sodium bicarbonate until pH of
blood became normal
E. Intubation and ventilator to maintain
the respiratory

17. A 36-year-old male truck driver is referred to you


for evaluation of recurrent urolithiasis. The patient
has passed more than 20 calcium oxalate stones
since
age
18.
Previous
therapy
with
hydrochlorothiazide and cellulose phosphate has
been ineffective. Physical examination is normal.
Laboratory finding: Serum creatinine 1.0 mg/dL,
Serum uric acid 8.9 mg/dL, Urine pH 5.0
Radiographs of the abdomen show smooth renal
silhouettes measuring 13.6 cm on the right and
14.2 cm on the left, and no radioopaque at kidney
region. From sonography findings multiple
hyperechoic shadow from both kidney with
measure about 2 5 mm.
Which of the following treatment is the most
effective for this patient?
A. Dietary sodium restriction
B. Dietary calcium supplementation
C. High fluid intake and long-term urine
acidification
D. wHigh
fluid
intake
and
allopurinol(untuk
asam
urat=radiolucent)
E. Shock wave lithotripsy

18. A 75 year old man had urine incontinence since 3


months ago. He had been diagnosed dementia by
the doctor since 6 months ago. On examination he
had no prostate problems. What is the cause of his
urinations problem?
A. Impaired frontal inhibition
B. Impaired pontine inhibition
C. Sympathetic system dysfunction
D. Parasympathetic system dysfunction
E. Sensoric system dysfunction
Yg inhibit hrusnya pons? Brain stimulate pundendal
trusan. Dementia yg rusak frontal brainnya. Bkn
frontal inhibition , tp frontal stimulation hrusnya.
19. A 44 year old man was hospitalized for systemic
fungal infection. He was on Amfoterisin B
treatment for 7 days. On examination, he is
conscious, BP 120/80 mmHg, pulse 80 x/min, RR
16 x/min, temperature 36,7 C. Lab: Hb 13 gr%,
HT 37%, WBC 8600, Platelet 340.000. Ureum 91,
creatinin 2, 0. Volume urine 800 cc/24 jam
What is the patognomonic laboratory finding in this
case?
A. Increased osmolality
B. Leucocyturia
C. Muddy brown cast
D. Erythrocyte cast
E. Haemoglobinuria
20. A 24-year-old woman went to a physician because
pain during urination and frequency. On
examination, she had has suprapubic pain, but no
fever. Her urine was cloudy and shows
microscopic evidence of erythrocytes, pyuria, and
gram-positive bacteria. In the past medical history,
she never had any symptoms like this one. Which
organism is most likely responsible for this
patients illness?
A. Klebsiella pneumonia
B. Pseudomonas aeruginosa
C. Escherichia coli
D. Proteus mirabilis
E. Staphylococcus saprophyticus
21. A 55-year-old male complains hematuria at the
physicians office for 2 weeks. A cystoscopic
examinations showed a 2 cm,exophytic,friable
mass on the right bladder dome.
A biopsy of this lesion show findings most
consistent with a/an
A. Papilloma
B. Verrucous carcinoma
C. Transitional carcinoma
D. Squamous carcinoma
E. Adenocarcinoma
22. A 50-year-old male presents with a month long
history of flank pain and hematuria.An USG
abdominal results a 3 cm bosselated mass,
protrudes from
cortical are
in the right
kidney.Cytologic examination reveals malignant
cells.
What is most likely biopsy finding for this patient?

A.
B.
C.
D.
E.

Wilms tumor
Renal Cell Carcinoma
Transitional carcinoma
Squamous carcinoma
Undifferentited carcinoma

23. A 60-year-old male experiences the onzet of


headache, naussea ,vomitting fo 1 month.
Physical
examination
reveals
hypertention,
subfebril. Pyelogram showed the affected kidney
assymmetrically contracted with deformity of the
calyceal system. Gross of the affected kidney
showed contracted and has irregular granular
surface.The parenchyma is atrophic and replace
by fibrosis. Microscopic
showed glomerular
sclerosis, hyalinization and atrophi in cortical area.
Another
areas
showed
fibrosis,
chronic
inflammatory cells with limpocytes agregation.
What is the most likely lesions occur in this
kidney?
A. Renal cell carcinoma
B. Chronic pyelonephritis
C. Acute nephritis
D. Nephrotic syndrome
E. Diabetic nephropathy
24. A 5-year-old boy child who had a palpable
abdominal mass brought to hospital by his mother.
The child later had abdominal distention from
bowel obstruction. An USG reveals a 6 cm left
renal mass with necrosis and hemorrhage.
Microscopic
showed embryonal tissue with
abortive of tubular and glomerular structures.
What is the diagnosis ot this patient?
A. Teratoma
B. Retinoblastoma
C. Wilmstumor
D. Hanarthoma
E. Hemangioma
25. A 62-year-old woman presents with hematuria and
left flank pain. Physical examination reveals
costovertebral angle tenderness. Renal ultrasound
shows marked left hydronephrosis. Serum
creatinine and CBC are normal.
Which of the following diagnostic test is the most
relevant?
A. Renal biopsy
B. Renal arteriogram
C. Palpation for inguinal lymph nodes
D. Antinuclear antibody test
E. IVP
26. A 34-year-old man comes with 3-day history of low
back pain, urinary hesitancy, and pain with
urination. He has not had any urethral discharge
or recent sexual contacts. His temperature is
38.2C,Z
27. A 25-year-old man has been crashed by a truck.
The doctor suspects that his kidney is rupture, and
then he sent the patient to have IVP.

What is the most likely the appearance of renal


parenchymal rupture in IVP?
A. The kidney shows non function
B. Ekstravasation (urine bocor )
C. Hydronephrosis
(urine akumulasi di
renal pelvis)
D. Normal nephrogram
E. Dilatation of calyx
28. A 50-year-old man comes to the clinic with
complain of abdominal colic. The result of IVP
demonstrates ureteric dilatation.
What is the most likely problem?
A. Stone in the pelvicureter junction
B. Kidney cyst
C. Stone in the vesicoureter junction
D. Renal cell Ca stage I
E. Acute inflammation
29. A 24-year-old woman went to a physician because
of pain during urination. On examination, she had
has left flank tenderness and low-grade fever. Her
urine was cloudy and shows microscopic evidence
of erythrocytes, pyuria, and gram-positive bacteria.
In the past medical history, she never had any
symptoms like this one.
Which organism is most likely responsible for this
patients illness?
A. Klebsiella pneumonia
B. Pseudomonas aeruginosa
C. Escherichia coli
D. Proteus mirabilis
E. Staphylococcus saprophyticus
30. Which urine specimen is most appropriate for the
culture examination?
A. Suprapubic aspiration
B. Mid stream
C. Straight catheter
D. Indwelling catheter
E. 24 hours urine collection
31. A 30-year-pregnant woman was complaining fever,
pain when urinating, blood in her urine, and severe
lower back pain. The physician submits her urine
for culture; the next day, the laboratory reports an
5
organisms growth count of greater than 10 /ml of
urine. The organism grew well on the blood agar
and producing hemolytic colonies.
Which important virulence factor for the organism
is most likely responsible for this patients illness?
A. Capsule
B. P-fimbriae (Ecoli)
C. Endotoxin
D. Phospolipase C
E. Lipooligopolysaccharide
32. A 30-year-old man patient complained of a sharp
pain in his loin radiating down to his perineum,
accompanied by hematuria. He reported that he
had a few days of urinary urgency, frequency, and
burning when urinating. On examination, his
temperature was 37.5C; other vital sign were

normal; and he appears to be in pain. A urinalysis


shows erythrocytes, leukocytes esterase and a
high pH.
Which antigenic composition is most likely
responsible for this illness?
A. Hemolysin (proteus)
B. Catalase
C. Fibronectin
D. Opa protein go
E. Cord factor tb
33. A 67-year-old man with an episode of acute urine
retention was catheterized. Three days later, he
developed fever and suprapubic pain. Culture of
urine revealed a thin film of bacterial growth over
the entire blood agar plate, and urease test was
positive.
Which of the following is most likely organism to
cause this infection?
A. Escherichia coli
B. Staphylococcus sapropyticus
C. Proteus mirabilis
D. Enterococcus faecalis
E. Morganella morgagni
34. A 34-year-old woman presents with pain during
urination and frequency. On examination, she had
has suprapubic pain, but no fever. Her urine was
cloudy and shows microscopic evidence of
erythrocytes, pyuria, and bacteria. The urine has a
high pH. Which of the following bacterias strategy
is responsible for this condition?
A. Destroy blood vessel in the kidney
B. Exhibits swarming motility
C. Ferments many sugars
D. Produce a potent urease
E. Secretes many exotoxins
35. IVP image of a new born baby shows that left
kidney tremendously enlarged and clearly
demonstrates intra-parenchyma mass. What
abnormality could possibly occur?
A. horseshoe kidney
B. pelvic kidney
C. unilateral renal agenesis
D. polycystic kidney (ada massa)
E. pancake kidney
36. Oligohydramnios can be related to malformation in
kidney. Why can the amount of amniotic fluid
determine such malformation?
A. baby drinks the amniotic fluid
B. baby cannot drink amniotic fluid
C. amniotic fluid mostly contains urine
D. little amount of amniotic fluid causes
the pelvic malformation
E. little amount of amniotic fluid disturbs
the kidney development
37. A new born baby boy has a malformation in
genitourinary appearance. When he urinates, the
urine comes out from large area below the urethra

opening. What process could possible failed during


the development of genitourinary process?
A. closure of genital fold
B. closure of genital opening
C. closure of genital tubercle
D. closure of urogenital sinus
E. closure of Wollfian duct
38. Cryptorchid condition results in germ cells failed to
multiply, and then die and only in Sertoli cells exist
in seminferous tubules. What is the development
process the main cause of cryptorchid?
A. inguinal hernia
B. failure of testicular decent
C. failure of gubernaculums regression
D. fusion of the tunica albuginea and
peritoneum
E. guevedoces
39. A 30-year-old man had nausea, vomiting, and
severe colicky right flank pain radiating into the
thigh for 4 hours. He is afebrile. There is right
costovertebral angle tenderness. Urinalysis shows
numerous RBCs and no bacteria.
Which of the following is the most likely diagnosis?
A. Acute glomerulonephritis
B. Bacterial cystitis
C. Benign prostatic hyperplasia
D. Urolithiasis
E. Renal cell carcinoma
40. A previously healthy 72-year-old man comes to the
physician because of decreased urinary output
during the past 2 days; he has had no urinary
output for 8 hours. Examination shows suprapubic
fullness and enlarged prostate.
Which of the following is the most appropriate next
step in management?
A. PSA examination
B. Ultrasonography of the kidneys
C. CT scan of the abdomen
D. Bladder catheterization
E. Intravenous pyelography
41. An otherwise healthy 20-year-old woman is given
sulfamethoxazole to treat a bladder infection.
Three days after beginning the antibiotic regimen,
the patient has moderately severe jaundice and
dark urine. Pain with urination and a low-grade
fever have resolved. Her hematocrit is 20%.
Substantial numbers of erythrocytes contain Heinz
bodies. Her condition worsens until day 6 of
antibiotic therapy, when it begins to resolve.
Symptoms are completely gone by day 9 of
continued antibiotic therapy.
Which of the following conditions is the most likely
explanation for these findings?
A. Aplastic anemia
B. Generalized cytochrome-b5 reductase
deficiency
C. Glucose-6-phosphate dehydrogenase
deficiency
D. Pyruvate kinase deficiency

E.

Steven Johnson Syndrome

42. A 61-year-old man comes to the physician


because of a 3-month history of episodes of
headache, palpitation, and excessive sweating. He
has had a 10-kg (22-lb) weight loss during this
period. While being examined, during an episode
his blood pressure is 210/110 mm Hg. Physical
examination shows no other abnormalities. Urine
studies
show
increased
catecholamine
concentrations. A CT scan of the abdomen is most
likely to show a mass in which of the following
locations?
A. Adrenal glands (aldosterone dan
catecolamin dan adrenalin)
B. Ureter
C. Urethral
D. Bladder
E. Right kidney
43. A 6-year-old boy has a large intra-abdominal mass
in the midline just above the symphysis pubis.
During an operation, a cystic mass is found
attached to the umbilicus and the apex of the
bladder. Which of the following is the most likely
diagnosis?
A. Hydrocele
B. Meckel cyst
C. Meckel diverticulum
D. Omphalocele
E. Urachal cyst
44. A 69-year-old man with 82 kg is referred to you
with fatigue and laboratory result shows Hb 7 g/dL,
ureum 55 mg/dL, and creatinin 5.4 mg/dL. He has
not seen a physician for several years.
Which finding would help you decide that this
condition is chronic?
A. Low serum calcium
B. Low serum bicarbonate
C. Protein detected in urinalysis
D. Broad waxy casts in urinalysis
E. Elevated serum phosphorus
45. A 52-year-old woman with longstanding type II
diabetes is referred with microalbuminuria and
normal serum creatinine. Her weight is 85 kg and
height is 160 cm. Blood pressure is 150/100
mmHg.
Which of the following measures should you
advise to prevent progression to overt
nephropathy?
A. Tight glucose control
B. High-protein and low carbohydrate
diet
C. Blood pressure control
D. Regular diuretic
E. Increased water intake
46. A 20-year-old man presents with malaise, nausea,
and decreased urine output. He was previously
well and his physical examination is normal except

for an elevated jugular venous pressure (JVP) and


pericardia rub. His electrolytes reveal acute renal
failure (ARF).
Which of the following findings on the urinalysis is
most
likely
in
keeping
with
acute
glomerulonephritis?
A. Granular casts (dan epithelial = ATN)
B. Erythrocyte casts
C. Proteinuria
D. Hyaline casts (prerenal)
E. White blood cell casts
47. A 35-year-old women presents with nausea and
vomiting of 2 days duration. She is not on any
medications and was previously well until now. Her
physical examination is normal except for postural
drop in her blood pressure from 110/80 mmHg
supine to 90/80 mmHg standing. Her serum
electrolytes are Sodium 130 mEq/L, Potassium 3
mEq/L, Chloride 90 mEq/L, bicarbonate 30 mEq/L,
urea 50 mg/dL, and creatinine 0.8 mg/dL.
Which of the following electrolytes in the case
above is most likely to be filtered through the
glomerulus but unaffected by tubular secretion?
A. Potassium
B. Urea
C. Creatinine
D. Bicarbonate
E. Sodium
48. A 68-year-old man came seeing a general
practitioner with a swelling throughout the body.
He also felt urine volume decreases and the urine
color was red. He denied any pain on urination or
in any part of his body, on physical examination
costovertebrae angular test negative. Blood test
results obtained ureum 55 mg/dL (normal 20-40
mg/dL) and creatinine 1.7 mg/dL (normal <1.1
mg/dL). Obtained urine lab protein 2 + and 50-60
red blood cells.
Which of the following diuretics is the most
appropriate for treating the patient above?
A. Acetazolamide
B. Furosemide
C. Hidrochlorothiazid
D. Spironolactone
E. Triamteren
49. A 10 month pregnant woman complained lower
abdominal pain, and suffering from dysuria for 48
hours. Patient has mild fever, urine analysis
showed 1+ protein, but did not find any glucose,
and blood. Gram stain shows a gram negative
bacteria.
Which of the following drugs is the most
appropriate for this patient condition?
A. Chloramphenicol
B. Cefixime (ibu hamil), ga bleh
quinolone
C. Cephadroxil
D. Cotrimoxazole
E. Tetracycline

50. A 6-year-old girl, brought by her mother to the


clinic with complaints of swelling that begins from
both of eyelid throughout her body. Blood tests
obtained: albumin 2 g / dL, total cholesterol 250
mg / dL. On examination of urine obtained protein
+3. Which of the following is the most likely
diagnosis for the case above?
A. Nephritic syndrome
B. Nephrotic syndrome
C. Acute renal failure
D. Chronic renal failure
E. Glomerulonefritis
51. According to the case above, which of the
following is the most likely pathophysiology that
caused of swelling?
A. Infection
B. Hypercholesterolemia
C. Hypoalbuminemia
D. Proteinuria
E. Water retention
52. A 39-year-old woman with a history of recurrent
urinary tract infections and now suffering that
infection. Culture examination of urine samples
showed Escherichia colli as the cause of infection.
Patients were given doses of ciprofloxacin in the
treatment and symptoms of infection disappear.
Which of the following is the most correct process
or enzyme that inhibited by the drug?
A. Cell membrane synthesis
B. Cell protein synthesis
C. Dehidrofolate sintetase
D. DNA gyrase
E. DNA polimerase
53. A 74 years old man was admitted to the hospital
for swollen ankle, hands and eyelids. Her skin was
pale, damp and cool. The patients vital signs
were: blood pressure 170/90 mmHg, heart rate
100 beats/min, deep respiration 18 breath/min,
and temperature 37.8C. The patient had no urine
output and the laboratory report showed that the
patients blood urea nitrogen [BUN], creatinine,
potassium and phosphate level s was all higher
than normal. A chest x-ray showed white, fluffy
patches that spread outward from the hilar areas
to the peripheral border of both lungs. Her left
ventricle appeared moderately enlarged. Which of
the following is the most likely caused of an altered
renal function in this case?
A. Elevated BUN level
B. No urine output
C. Heart failure
D. Pulmonary disease
E. Infection
54. A 50-year-old woman went to a Puskesmas
complaining of edema in both legs which became
worse along the day. Her Blood pressure is 160/95
mmHg, pulse rate 70/min. She also had a history
of an ischemic stroke. Laboratory test are,fasting
blood sugar 150 mg/dL (N: <121 mg/dL), serum

creatine 2.1mg/dL (N: <1.1 mg/dL), ureum


100mg%, urinary protein ++. (CKD)
Which of the following is the most appropriate drug
should be given to the patient in order to prevent
secondary stroke attack?
A. Propranolol
B. Captopril
C. Reserpin
D. Hydrochlorothiazide
E. Furosemid
55. A 63-year-old man, came to a clinic with of pain on
her knees, there are also looked swelling and
redness at her both of knees. She said that she
also suffering edema in both legs which became
worse along the day. Her Blood pressure is
180/100 mmHg, pulse rate 90/min. Laboratory test
result: Hb: 10.8 gram%, serum creatine 2.7mg/dL
(N: <1.1 mg/dL), ureum 80mg%, urinary protein
++. You are suggesting giving an aspirin (NSAID)
h3X500 mg with caution.
Which of the following is the most reason that you
give the drug with caution?
A. It is ineffective
B. It causes hypokalemia
C. It worsen anemia
D. its metabolite is nephrotoxic
E. It decrease kidney filtration
56. A 55-year-old male complained at the physicians
office of his hematuria for 2 weeks now. Urine
cytologic examination showed malignancy cells. A
cystoscopic examination found a 2 cm, exophytic,
friable mass on the right bladder dome.
A biopsy of this lesion will likely to show findings
that mostly consistent with,
A. Papilloma
B. Verrucous carcinoma
C. Transitional carcinoma
D. Squamous carcinoma
E. Adenocarcinoma
57. Which of the following risk factors is most likely to
have led to development of this lesion?
A. Diabetes mellitus
B. Recurrent urinary tract infection
C. Therapy with methicillin
D. Cigarette smoking
E. Tuberous sclerosis
58. A 50-year old male was present with a month-long
history of flank pain and hematuria. An USG
abdominal imaged a 3-cm bosselated mass which
protruded from cortical in the right kidney. The
cytologic examination revealed malignant cells.
A biopsy of this mass will likely to show findings
that mostly consistent with,
A. Wilms tumor
B. Renal Cell Carcinoma
C. Transitional carcinoma
D. Squamous carcinoma
E. Undifferentited carcinoma

59. On physical examination to a 60-year old male


revealed hypertension and subfebril. He has had
history of nausea, vomit and right lateral
abdominal pain for 2 months. Pyelogram showed
that the right kidney smaller than of the left,
asymmetrically contracted with deformity of the
calyceal system. The right nephrectomy was
performed which showed contraction and it has
irregular granular surface. The parenchyma is
atrophic and replaced by fibrosis. The microscopic
showed glomerular sclerosis, hyalinization and
atrophy in cortical area. Another area showed
fibrosis, with lymphocytes aggregation.
Which of the following lesions could likely to
happen in this kidney?
A. Renal cell carcinoma
B. Chronic pyelonephritis
C. Acute nephritis
D. Nephrotic syndrome
E. Diabetic nephropathy
60. A 5-year-old boy child who had a palpable
abdominal mass was brought to hospital by his
mother. The child later had abdominal distention
from bowel obstruction. An USG revealed a 6cm
left renal mass with necrosis and hemorrhage.
The microscopic showed embryonal tissue with
abortive of tubular and glomerular structures.
The depicted lesion will most likely be,
A. Teratoma
B. Retinoblastoma
C. Wilmstumor
D. Hanarthoma
E. Hemangioma
61. A 45 years old man, came to the hospital. And told
the doctor that he had episodic severe pain in the
left abdomen and radiated into his flank and
sometimes into his testical (ipsilateral). From USG
the doctor found 6 mm stone in his urinary tract.
What location is the most possible for the
obstruction?
A. Upper ureter
B. Middle ureter
C. Lower ureter
D. Vesicae urinariae
E. Urethra
62. If the patient felt the pain radiated from left
abdomen to the testicle cause of a stone in his
ureter, what level of sympathetic fibers is the most
responsible for ureter?
A. T 7-8
B. S 2-4 (parasimpatik)
C. T 11-12 (simpatik bladder)
D. L1-3
E. L4-5
63. A 50-year-old man came to his doctor and told the
doctor that for the last 2 months, he felt burning on
urination and the color was dark. The doctor told
him to check his urine. And the result:

pH :7 (N : 4.5 8) , WBC: 15-20/HPF (N : 02/HPF) ; RBC : 5-10/HPF (N: 0/HPF ). Urine


sediments show crystals with cuboid shape. (ca
oksalat)
What is the most likely metabolic abnormality?
A. Hyperoxaluria
B. Hypercalciuria
C. Hypercitraturia
D. Hyperuricuria
E. Renal tubular acidosis
64. A 42-year-old man came to the hospital and said
that his urines color is red and he saw vermiform
clot. From his statement, could you tell the
possible source of his problem?
A. Ureter
B. Urethra
C. Bladder
D. Prostate
65. A 50-year-old man is hospitalized for acute
myocardial infarction. He has decreased cardiac
output with hypotension requiring multiple pressor
agents. His urine output drops over the next 3
days. His serum urea nitrogen increases to 59
mg/dL, with creatinine of 2.9 mg/dL. Urinalysis
reveals no protein or glucose, a trace blood, and
numerous hyaline casts. Five days later, he
develops polyuria and his serum urea nitrogen
declines.
Which of the following pathologic findings in his
kidneys is most likely to have caused his
azotemia?
A. Patchy tubular necrosis
B. Fusion of podocyte foot processes
C. Glomerular crescents
D. Hyperplastic arteriolosclerosis
E. Mesangial
immune
complex
deposition
66. A 26-year-old diabetic woman is seen in the ER for
sore throat. Rapid strep test is positive for
streptococcal pharyngitis and she was started on
ampicillin 500 mg four times a day. Three days
later, she develops hematuria associated with a
low grade fever. On physical examination, she has
a maculopapular rash and a temperature of 38 C.
Laboratory studies show: serum creatinine 3.6
mg/dl, WBC 8,700 with 56% PMN, 25% lymphs,
3% monos and 15% eosinophils. Urinalysis: pH
6.2, protein 2+, blood 3+, 65 RBCs/HPF, 20-30
WBCs/HPF, 3-4 WBC casts/HPF. ASTO value is
increased.
What is the most likely diagnosis?
A. Diabetic nephropathy
B. IgA nephropathy
C. Acute interstitial nephritis
D. MAcute pyelonephritis
67. A 40-year old man with arthritis has been using
ibuprofen (nonsteroidal anti-inflammatory) every 6
hours. He developed renal failure associated with
nephrotic range proteinuria.

What is the most likely glomerular lesion in this


patient?
A. Membranoproliferative
glomerulonephritis
B. Minimal change disease
C. IgA nephropathy
D. Diabetic nephropathy
E. Focal segmental glomerulosclerosis
68. A 4-year-old-boy was brought to hospital with
puffiness around the eyes, especially in the
morning and swelling over the legs. His mother
also complaint he has foamy urine. Urine sample
2
shows protein ++++ (>3.5 per 1.73 m per 24
hour), albumin level 1.5g/dL (normal=3.5-5g/dL).
What is the most likely underlying mechanism in
this patient?
What is the principal management in this patient?
A. Albumin 0,5-1 g/kg, iv, follow with
furosemide 1-2 mg/kgBB/iv
B. Prednisone 40 mg/m2/day
C. Prednisone 60 mg/m2/day
D. Reduce sodium intake
E. Alkylating agent
69. A 5 day old baby was hospitalized due to indirect
hyperbilirubinemia. On the third day hospitalization
he had fever and then he was given amoxicillin
(beta lactam yang bikin AIN) and gentamycin
intravenously. Three days later his urine output
diminished and the laboratory test revealed urea
nitrogen 25 (3-12) mg/dL, creatinine 0.9 (0.2-0.4)
mg/dL.
What is the most likely diagnosis in this patient?
A. Urinary tract infection
B. Sepsis
C. Acute renal failure
D. Chronic renal failure
E. Acute tubular nephritis
A 3 year old boy was brought to emergency unit
Karawaci Hospital with severe diarrhea since three
days ago. On arrival he is not fully alert. His vital sign
are as follows: blood pressure is 80/60 mmHg, pulse is
not palpable, respiratory rate was 36 breaths per
o
minute, and body temperature was 38.4 C. On
physical exam there is sunken anterior fontanel, dry
mucous membranes, sunken eyes, lack of tears, poor
skin turgor, and the capillary refill more then 2
seconds. The daily urine volume is less than 400 mL.
70. In this patient, the tubular epithelial cells are
particular susceptible to ischaemic injury due to:
A. Little charged surface for reabsorption
B. Low metabolic rate and oxygen
consumption
C. Decreased levels of salt reaching the
distal tubules
D. Loss
of
polarity
that
cause
redistribution of membrane proteins
E. Passive transport systems for ions
and organic acids

71. The laboratory test in this patient reveal


haemoglobin 15 g/dL( 11.5-15.5 g/dL), hematocrit
3
46%
(35-45%), WBC
7,000/mm
(5,5003
3
15,500/mm ),
platelets
count
260,000/mm
3
(150,000-400,000 mm ), urea nitrogen 40 mg/dL
(5-18 mg/dL), and creatinine 1.7 mg/dL (0.3-0.7
mg/dL).
The most likely diagnosis in this patient is:
A. Acute tubular nephritis
B. Prerenal acute kidney injury
C. Intrinsic acute kidney injury
D. Postrenal acute kidney injury
E. Chonic renal failure

76. A 64-year-old male presents with hematuria.


Examination shows a flank mass and he has
elevated hemoglobin. Pathology of the surgical
specimen shows a clear cell renal cell carcinoma.
What is the cell of origin of this tumor?
A. Glomerulocyte
B. Glomerular endothelium
C. Glomerular epithelium
D. Proximal tubular epithelium (clear ell n
papilla)
E. Histiocyte

A 10 year old girl was brought to your clinic with disuria


since 5 days ago. On physical exam she is healthy
with no significant findings. From urine culture is found
Eschericia coli with CFU 100,000.
72. What is the most likely diagnosis in this patient?
A. Acute pyelonephritic
B. Reflux nephropathy
C. Uncomplicated urinary tract infection
D. Complicated urinary tract infection
E. Chronic urinary tract infection

77. A 32-year-old female presents with headaches.


Investigation reveals hypertension (180/110
mmHg) and very high level of aldosterone
presumed due to a tumor. What of the following is
characteristic of this condition?
A. Elevated serum sodium levels
B. Elevated serum potassium levels
C. Elevated urinary cortisols
D. Elevated urinary bicarbonate

73. Factors unfavorable to bacterial growth in this


patients urine is:
A. A low pH
B. A low concentration of urea
C. The absence of organic acids from a
diet
D. A low osmolality
E. The
presence
of
normal
microorganism
74. An 8 years old boy presents with decreased urine
output, hematuria, high blood pressure, proteinuria
add rising creatinine. One week ago he had a
fever and sore throat. A renal biopsy is carried out
reveals diffuse proliferative glomerulonephritis.
Electron microscope will show abnormal deposits
in what location?
A. Between basement membrane and
endothelial cells of the glomeruli
B. Between the basement membrane
and epithelial cells of the proximal
tubule
C. Between the basement membrane
and epithelial cells of the glomeruli
D. Within the juxtaglomerular apparatus
75. A 45 years old female has a long history of poorly
controlled diabetes. She presents now with
edema, hypertension and massive proteinuria,
hematuria and hyperlipidemia. Renal biopsy shows
hyaline arteriosclerosis of the arterioles. What
changes will be present in the glomeruli?
A. Diffuse proliferative glomerulonephritis
B. Membranous
proliferative
gomerulonephritis
C. Membranous glomerulosclerosis
D. Focal segmental glomerulosclerosis
E. Nodular glomerulosclerosis

Distal (klomoford, oncodi)

78. An 18-year-old male presents to emergency


following a motorcycle accident. There is
significant blood loss. In the ER, his BP is 90/60
mmHg and his pulse is 120 x/min. What
physiological response occurs in the kidney?
A. Decreased urine osmolality
B. Reduced release and action of ADH
C. Beta adrenergic stimulation of the
vasculature
D. Vasoconstriction of the renal arterioles
E. Reduced aldosterone production and
sodium loss
1. Mr. Edi, 42 year old male, comes to the Siloam
hospital with muscular weakness, mild headache
after eating salted fish. Previously he never eats
salted fish. Usually he drink only small amount of
water. He said that usually his blood pressure is
100/70 mmHg. His blood pressure 140/90 mmHg.
The doctor sends him to the laboratory. The result
shows serum sodium is 150 mEq/L, potassium is
3.5 mEq/L, glucose 150 mg/dl.
In this condition, what will the kidney do?
A. Increase GFR
B. Increase obligatory water reabsorption
C. Increase facultative water reabsortion
D. Increase renin secretion
E. Increase aldosteron secretion
2. To cover the above function, what mechanism is
used?
A. Increase cardiac output
B. Increase frequency of the heart
C. Increase secretion of renin
D. Increase secretion of antidiuretic hormone
E. Increase secretion of aldosteron
.
3. Which factor support the above mechanism?
A. Filtration rate of the glomeruli

B. Active transport of the tubular cell


C. High osmolality of the interstitium of medulla
D. Obligatory water reabsortion
E. Osmolality of the filtrate in distal convoluted
tubule
4. If the urinalysis is also done, what will you see about
the glucose in urine?
A. positive ++++
B. positive +++
C. positive ++
D. positive+
E. negative
5. In a kidney laboratory, using a micropipette, we can
measure the hemodynamic pressure of afferent
arteriole, pressure in Bowman capsule, and osmotic
pressure in glomerular lumen. If the pressure in
afferent arteriole is 40 mmHg, in Bowman capsule 1
mmHg, the osmotic pressure of glomerulus is 30
mmHg, the effective filtration pressure in glomerulus is:
A. 71 mmHg
B. 69 mmHg
C. 11 mmHg
D. 9 mmHg
E. 9 mmHg
6. A patient taking an oral diuretic for about six
months, visits your clinic with elevated fasting and
postprandial blood glucose levels. You check the
patients HbA1C and find it is elevated compared
with normal baseline values obtained 6 months
ago. You suspect the glycemic problems are
diuretic-induced. Which of the following drug is the
most likely cause?
A. asetazolamide
B. amiloride
C. chlorothiazide
D. spironolactone
E. triamterene
7. Your patient, a 55 year-old man, with heart failure,
unacceptably low cardiac output, and intense
reflex-mediated sympathetic activation of the
peripheral vasculature that is attempting to keep
vital organ perfusion pressure sufficiently high. The
patient is edematous, and has ascites, because of
the poor cardiac function and renal compensations
for it. Which of the following drug should be
avoided in this patient?
A. amiloride
B. ethacrynic acid
C. hydrochlorothiazide
D. mannitol
E. spironolactone
8. A 63-year-old-man, with a history of heart failure
and edema fails to respond adequately to
maximum recommended dosages of
chlorthalidone.
Which of the following is the most likely
appropriate for restoring the diuretic response?

A. add with hydrochlorothiazide


B. add with metolazone
C. replace with furosemide
D. replace with hydrochlorothiazide
E. increasing the dose of
chlorthalidone
9. A 75-year-old woman with hypertension is being
treated with thiazide. Her blood pressure is
decrease to 120/76 mm Hg. After several months
on the medication, she complains of being tired
and weak. Which of the following substance in
analysis of blood indicates low values?
A. calcium
B. uric acid
C. potassium
D. sodium
E. glucose
10. Your patient, a 45-year-old man, usually lives near
the beach, is planning to have a vacation that
includes a short hike to the top of Mount Everest
(altitude approx. 29.000 feet above sea-level).
You are concern about mountain sickness. He
has no other significant medical conditions and
takes no other drugs that would interact with the
drug you will prescribe for his trip.
Which of the following drug you recommend?
A. acetazolamide
B. amiloride
C. bumetabide
D. furosemid
E. spironolactone
11.

A patient with tuberculosis develops bright


orange-red urine after he drank his antituberculosis drugs and calls his physician in a
panic because he is afraid he is bleeding into the
urine. The patient has no other urinary tract
symptoms. Which of the following medication is
the most likely to produce this side effect?
A. Ethambutol
B. Isoniazid
C. Pyridoxine
D. Rifampicin
E. Streptomycin

12. A 26 year-old young man presents with the


symptoms of gonorrhea. This condition is often
associated with infection due to Chlamydia
trachomatis.
Which of the following quinolones would be the
best choice for treating him?
A. ciprofloxacin
B. nalidixic acid
C. norfloxacin
D. levofloxacin
E. ofloxacin
13. A jaundiced 1-year premature infant with elevated
free bilirubin is seen in the premature baby
nursery. The mother had received antibiotic

10

combination for a urinary tract infection (UTI) 1


week before delivery.
Which of the following is the most likely cause of
the babys kernicterus ?
A. cefixime
B. amoxicillin
C. azithromycin
D. erythromycin
E. cotrimoxazole
14. A 30 year-old woman with a history of recurrent
urinary tract infection. Since 5 days ago, she felt
dysuria, urgency, and frequency. Culture of urine
sample indicated that the offending organism is
Escherichia coli. She receives ciprofloxacin and
the symptoms disappear.
Which of the following process is inhibited by
ciprofloxacin?
A. cell-wall synthesis
B. folic acid synthesis
C. protein synthesis
D. topoisomerase II
E. DNA polymerase
15. A 30 year old man with motorcycle accident comes
to the emergency. In the
physical examination, the doctor found that his left
kidney has severe injured.
And the doctor suggested operating his kidney.
During the removal of a patients kidney,
Which of the following structure is the most anterior
within the renal sinus?
A. Renal arteries
B. Renal vein
C. Major calyx
D. Minor calyx
E. Renal pelvis
16. You wish to examine the hilum of the left kidney
during surgery.
Which of the following structures must be elevated?
A. Stomach
B. Suprarenal gland
C. Ascending colon
D. Duodenum
E. Liver
17. To elevate the kidney within the renal fascia and
the perirenal fat, which of the
following muscles must be reflected or incised from
the fascia?
A. Diaphragm
B. Psoas
C. Quadratus lumborum
D. Transverses abdominis
E. Iliacus
18. An 18-year-old man is noted to have probable
testicular cancer. He undergoes
surgery. After incising the scrotum, the surgeon
contemplates the approach
to the parenchyma of the testes.

Which layer is the most near to the testicular


parenchyma?
A. Fascia spermatica externa
B. Tunica albuginea
C. Fascia sprematica interna
D. Scarpa fascia
E. Tunica dartos
19. A 18-year-old man is noted to have dysuria for
several days, and the doctor diagnose that she suffers
from urinary tract infection.
Which of the following structure of the urinary tract
is the most vulnerable to
get infection?
A. Kidney
B. Ureter
C. Urinary bladder
D. Urethra
E. Adrenal gland
20. A 70 year old man came to the emergency
department of Karawaci Hospital with
complaint of incomplete urination since 6 months
ago. The urologist performed
rectal examination and found an enlargement of
his prostate.
Which of the following structure is affected by the
enlargement?
A. urethra pars membranacea
B. urethra pars spongiosa
C. sphichter urethra interna
D. urethra pars prostatica
E. orificium urethra externa
A 4-year-old-boy was brought to hospital with puffiness
around the eyes, especially in the morning and
swelling over the legs. His mother also complaint he
has foamy urine. Urine sample shows protein ++++
2
(>3.5 per 1.73 m per 24 hour), albumin level 1.5g/dL
(normal=3.5-5g/dL).
21. What is the principal management in this patient?
F. Albumin
G. Furosemide
H. Prednisone
I. Diet restriction
J. Cyclosporin
22.

During 4 weeks-treatment, the urine sample


shows protein +.
What is your conclusion in this patient?
A. Initial responder
B. Late responder
C. Drug resistant
D. Drug dependent
E. Remission

A 13 year-old-girl was brought to emergency unit with


seizure. Her mother complaint she had fever 2 weeks
ago, but has recovered. Now she has swelling around
the eyes and legs and she has dark urine. From
physical exam, she is unconscious. Her blood
pressure is 180/120 mmHg. Urine sample shows

11

erythrocytes 20/hpf and protein +. Albumin level within


normal limit.
23. What other laboratory findings to confirm the
diagnosis in this patient?
F. Decreased anti streptolisin O (ASO),
increased DNase-B, decreased C3
G. Increased anti streptolisin O (ASO),
decreased DNase-B, decreased C3
H. Decreased anti streptolisin O (ASO),
increased DNase-B, increased C3
I. Increased anti streptolisin O (ASO),
decreased DNase-B, increased C3
J. Increased anti streptolisin O (ASO),
increased DNase-B, decreased C3
24. What is the most likely mechanism that underlying
this patients disease?
A. Streptococcal bacteria infection
B. An inflammation of the kidney tubules
C. Increased permeability of glomerulus
membrane
D. Small pores in the podocytes permit
erythrocyte loss
E. Heparan sulfate mucopolysaccharide is
low so erythrocyte cross the barrier
A 5 day old baby was hospitalized due to indirect
hyperbilirubinemia. On the third day hospitalization he
had fever and then he was given amoxicillin and
gentamicin intravenously. Three days later his urine
output diminished and the laboratory test revealed
urea nitrogen 25 (3-12) mg/dL, creatinine 0.9 (0.2-0.4)
mg/dL.
25. What is the most likely diagnosis in this patient?
F. Urinary tract infection
G. Sepsis
H. Acute renal failure
I. Chronic renal failure
J. Acute tubular nephritis
26. What is the most possible cause in this patient?
F.
G.
H.
I.
J.

Ischemia
Direct toxic injury
Complex immune response
Urinary obstruction
Drug hypersensitivity

27. What is the pathognomonic laboratory finding


(urinalysis) in this patient?
A. Increased osmolality
B. Proteinuria
C. Muddy brown casts
D. Erythrocyte casts
E. Haemoglobinuria
A 3 year old boy was brought to emergency unit
Karawaci Hospital with severe diarrhea since three
days ago. On arrival he is not fully alert. His vital sign
are as follows: blood pressure is 80/60 mmHg, pulse is
not palpable, respiratory rate was 36 breaths per

minute, and body temperature was 38.4 C. On


physical exam there is sunken anterior fontanel, dry
mucous membranes, sunken eyes, lack of tears, poor
skin turgor, and the capillary refill more then 2
seconds. The daily urine volume is less than 400 mL.
28. In this patient, the tubular epithelial cells are
particular susceptible to ischemic injury due to:
F. Little charged surface for reabsorption
G. Low metabolic rate and oxygen
consumption
H. Decreased levels of salt reaching the
distal tubules
I. Loss of polarity that cause redistribution of
membrane proteins
J. Passive transport systems for ions and
organic acids
29. The laboratory test in this patient reveal
haemoglobin 15 g/dL(11.5-15.5 g/dL),
hematocrit 46% (35-45%), WBC
3
3
7,000/mm (5,500-15,500/mm ), platelets count
3
3
260,000/mm (150,000-400,000 mm ), urea
nitrogen 40 mg/dL (5-18 mg/dL),
and creatinine 1.7 mg/dL (0.3-0.7 mg/dL).
What is the most likely diagnosis in this patient?
F. Acute tubular nephritis
G. Prerenal acute kidney injury
H. Intrinsic acute kidney injury
I. Postrenal acute kidney injury
J. Chonic renal failure
30. What is the most prior management in this patient?
A.
B.
C.
D.
E.

Bicarbonate
Acute Dialysis
Furosemide
Antibiotic
Fluid replacement therapy

A 10 year old girl was brought to your clinic with


dysuria since 5 days ago. On physical exam she is
healthy with no significant findings. From urine culture
is found Eschericia coli wih CFU 100,000.
31. What is the most likely diagnosis in this patient?
F.
G.
H.
I.
J.

Acute pyelonephritic
Reflux nephropathy
Uncomplicated urinary tract infection
Complicated urinary tract infection
Chronic urinary tract infection

32 .Which of the following factor is unfavorable to


bacterial growth in patients urine?
F. A low pH (5.5 or less)
G.
H.
I.
J.

A low concentration of urea


The absence of organic acids from a diet
A low osmolality
The presence of normal microorganism

33. Which of the following antibiotic is the most


appropriate for this patient?

12

A.
B.
C.
D.
E.

Ceftriaxone
Cefotaxime
Ceftazidime
Cotrimoxazole
Metronidazole

(C) It occurs in a minority of patients with bladder


cancer
(D) It commonly causes anemia
(E) It occurs primarily only in the initial phase of
the urinary stream
39.

A 55-year-old man refers to ICU after the procedure


open reduction of femur fracture and bleeding intra
operative. At the third day he became apatis, fever
38,5 Celcius with blood pressure 110/60 mmHg, pulse
55 x/min, respiratory 20x/min, and urine output
decreased to 10 cc/hour in 24 hours. At laboratory
examination findings creatinine 4 mg/dL, BUN 250
mg/dL, sodium 130 mEq/L, chloride 90 mEq/L,
potassium serum 7.5 mEq/L, blood sugar 200 mg/dL.
From blood gas analysis finding pH 7.1, pCO2 30
mmHg, pO2 90 mmHg,
HCO3 18 mEq/L. ECG examination finds widening of
QRS wave.
34. What is the most critical condition can cause life
threatening the patient?
(A) Nothing for awareness
(B) creatinine and BUN at very high level
(C) hyperkalemia
(D) metabolic acidosis
(E) Severe hyponatremia
35. What is the best choice of treatment for the clinical
situation for the patient?
(A) Food restriction with potassium composition
(B) Give K sparing diuretic
(C) Give regular Insulin 10 U in dextrose 50%
intravenous
(D) Give bicarbonate sodium intravenous
(E) Give forced diuretic
36. After the choice procedure, what is the next
treatment for the patient?
(A) Waiting for the general condition of the
patient became more better
(B) Give diuretic until the urine output will be
normally
(C) Hemodialysis immediately
(D) Give sodium bicarbonate until pH of
blood became normal
(E) Intubation and ventilator to maintain the
respiratory
37. According to RIFLE criteria, what is clinical
condition of the patient?
(A) Risk
(B) Injury
(C) Failure
(D) Loss
(E) End State Kidney Disease
38. Which of the following statements in true of
hematuria caused by bladder cancer?
(A) It is usually accompanied by discomfort and
painful voiding
(B) It is intermittent and painless

A 36-year-old male truck driver is referred to you


for evaluation of recurrent urolithiasis. The
patient has passed more than 20 calcium
oxalate stones since age 18. Previous therapy
with hydrochlorothiazide and cellulose
phosphate has been ineffective. Physical
examination is normal. Laboratory finding:
Serum creatinine 1.0 mg/dL, Serum uric acid 8.9
mg/dL, Urine pH 5.0
Radiographs of the abdomen show smooth renal
silhouettes measuring 13.6 cm on the right and
14.2 cm on the left, and no radioopaque at
kidney region. From sonography findings
multiple hyperechoic shadow from both kidney
with measure about 2 5 mm.
Which of the following treatment is the most
effective for this patient?
(A) Dietary sodium restriction
(B) Dietary calcium supplementation
(C) High fluid intake and long-term
urine acidification
(D) High fluid intake and allopurinol
(E) Shock wave lithotripsy
40. A 40-years-patient arrives at ER department with a
transthoracic gunshot wound and microscopic
hematuria.
What is the most appropriate supporting diagnostic
technique?
(A) Abdominal sonography
(B) Intravenous pyelography
(C) Immediate laparatomy
(D) Clinical observation on the hospital
ward
(E) Immediate CT with i.v contrast agents
41. You are asked to evaluate a 38-year-old woman
who has a ten-year history of bipolar disorder. She
has taken lithium carbonate for seven years. For
the past two years, she has had severe polyuria
(urine volume up to 10 L/24 hr). She refuses to
discontinue lithium therapy.
What is the most appropriate agent to reduce the
polyuria?
(A) Amiloride
(B) Chlorpropamide
(C) Furosemide
(D) Hydrochlorothiazide
(E) Acetazolamide
42. A man 45 years old come with bruise at lower
abdominal and pain. He is suspected with
bladder injury,
Which of the following statement is true about
bladder injuries?

13

(A) They are present 6% -10% in patients with


pelvic fracture
(B) More than 80% come with urine retention
(C) Delayed diagnosis is the rule
(D) Bladder contusions commonly result in
gross hematuria
(E) They are associated with a 50% rate of
urethral tear
43. A 51-year-old man, who is hospitalized for
treatment of Pneumocystis pneumonia, is found to
be HIV seropositive. There is no family history of
kidney disease or hypertension. Temperature is
38.1 C (100.6 F). Pulse rate is 102 per minute, and
respirations are 23 per minute. Urinalysis show
Protein 3+; 02 RBCs; 01 WBC/hpf, and Urine
protein is 6230 mg/24 hr. Ultrasonography
demonstrate large, echogenic kidneys.
What pathology feature is demonstrated in the
Kidney biopsy?
(A) Postinfectious glomerulonephritis
(B) Membranoproliferative
glomerulonephritis
(C) Membranous glomerulopathy
(D) Collapsing glomerulopathy
(E) Minimal change disease
44. A 36-year-old woman comes to the emergency
department because of severe, sharp right flank
pain of four hours' duration. During physical
examination, she cannot lie because of pain.
Temperature is 39.0 C (102.2 F), and pulse rate is
120 per minute. The right flank is tender to deep
palpation. Leukocyte count is 18,000/cu mm.
Serum creatinine is 2.0 mg/dL. Urinalysis shows
protein 1+; 1020 WBCs, 1020 RBCs/hpf.
Radiograph of the kidneys, ureters, and bladder
shows a 1.5-cm calcific density in the area of the
right mid-ureter. Both kidney silhouettes measure
approximately 12 cm pole-to-pole.
Ultrasonography shows a hydronephrotic right
kidney and contracted left kidney.
What is the most appropriate management for this
woman?
(A) Intravenous urography
(B) Intravenous ketorolac to relax the ureter
(C) Intravenous fluids that allow the stone to
pass
(D) Immediate percutaneous nephrostomy
(E) Emergency lithotripsy
45.

Normal micturition requires that the urinary


bladder and the urethral sphincter work together
as a coordinated unit to store and empty urine.
Which of the following is the effect of
parasympathetic system in controlling the
urinary bladder?
A. Relaxes external sphincter
B. Relaxes internal spinchter
C. Relaxes detrussor muscle
D. Contracts detrussor muscle
E. Contracts anal spinchter

46. The parasympathetic outflow in the spinal cord


occurs at levels:
A. T3,4
B. T 6-10
C. S1,2,3
D. S2,3,4
E. L1,2,3
47. A 75 year old man had urine incontinent since 3
months ago. He had been diagnosed dementia by
the doctor since 6 months ago. On examination he
had no prostate problems. What is the cause of his
urinations problem?
F. Impaired frontal inhibition
G. Impaired pontine inhibition
H. Sympathetic system dysfunction
I. Parasympathetic system dysfunction
J. Sensoric system dysfunction
48. A 40 years old woman, body weight 50 kg, came
to emergency unit with diarrhea (frequency 8
times/day) since 2 days ago. Her urine volume
was 150 cc/12hours. She was conscious, BP
85/60 mmHg, pulse 105 x/min, RR 20 x/min,
temperature 36,5 C. Laboratory results: Hb 12
gr%, Ht 40%, WBC 9000, Platelet 263.000, ureum
104, creatinine 3,7.
What is the most appropriate initial treatment for
the patient?
A. Give an antibiotic
B. Ringer lactate infusion
C. Give vasopressin
D. Glucose infusion
E. Oral rehydration
49. A 44 year old man was hospitalized for systemic
fungi infection. He was on Amfoterisin B treatmet
for 7 days. On examination, he conscious, BP
120/80 mmHg, pulse 80 x/min, RR 16 x/min,
temperature 36,7 C. Lab: Hb 13 gr%, HT 37%,
WBC 8600, Platelet 340.000. Urinalysis: RBC 01/HPF, WBC 2-5/HPF, protein (-), granular cast
(+). Ureum 91, creatinin 2, 0. Volume urine 800
cc/24 jam
What is the most likely diagnosis?
A. Acute Tubular necrosis
B. Acute Glomerular Nefritis
C. Acute Interstitial nephritis
D. Acute Renal Failure
E. Chronic Renal failure
50.

A young woman went to a physician because


pain during urination and frequency. On
examination, she had has suprapubic pain, but
no fever. Her urine was cloudy and shows
microscopic evidence of erythrocytes, pyuria,
and gram-positive bacteria. In the past medical
history, she never had any symptoms like this
one. Which organism is most likely responsible
for this patients illness?
F. Klebsiella pneumonia

14

G.
H.
I.
J.
51.

52.

Pseudomonas aeruginosa
Escherichia coli
Proteus mirabilis
Staphylococcus saprophyticus

Which urine specimen is most appropriate for


the culture examination for the patient above?
A. Suprapubic
B. Clean catch
C. Straight catheter
D. Indwelling catheter
E. 24 hours urine collection
A pregnant woman was complaining fever, pain
when urinating, blood in her urine, and severe
lower back pain. The physician submits her urine
for culture; the next day, the laboratory reports
an organisms growth count of greater than
5
10 /ml of urine. The organism grew well on the
blood agar and producing hemolytic colonies.
Which important virulence factor for the
organism is most likely responsible for this
patients illness?
A. Capsule
B. P-fimbriae
C. Endotoxin
D. Phospolipase C
E. Lipooligopolysaccharide

E. Ask him to drink while thirst only. Choose dry


foods.
55.

Protein recommendation:
A. No need to restrict protein
B. Use meat based protein not from the
legumes or beans
C. Limit protein high in uric Acid such as
beans, seeds
D. Limit protein 0,7 gram/kg BB

After a year he comes with weakness, nausea, muscle


cramp, and itchy. Physical exam: BW 52 kg, edema in
both legs and palpebras. Blood pressure 170/100
mmHg. Lab exam: Hb 7,6 g/dL, Ureum 196 mg/dL.
Creatinine 3,7 mg/dL. Uric Acid 11,8 mg/dL. Organic
Phosphorus 11 mg/dL, Calcium 3,4 mg/dL Sodium 147
mmol/L, Potassium 5,6 mmol/L.
56.

What is the most appropriate method to nourish


him:
A. Parenteral nutrition and nil by
mouth
B. Give a fruit juice and add sugar.
C. Give milk in enough calories.
D. Temporary no fruits and give
blended vegetable.

57. About calcium phosphorus balance in this patient:


53.

A male patient complained of a sharp pain in his


suprapubic radiating down to his perineum,
accompanied by hematuria. He reported that he
had a few days of urinary urgency, frequency,
and burning when urinating. On examination, his
temperature was 37,5C; other vital sign were
normal; and he appear to be in pain. A urinalysis
shows erythrocytes, leukocytes esterase and a
high pH.
Gram stain demonstrates negative coccobacili.
Which antigenic composition is most likely
responsible for this illness?
A. Hemolysin
B. Catalase
C. Fibronectin
D. Opa protein
E. Cord factor
A 45 years old man comes to Siloam Hospitals with
low back pain and painful sensation when urinating.
Anamnesis: he has a history of kidney stone. Physical
examination: pallor, fluctuating pain, blood pressure
140/90 mm Hg. Weight 54 kg, Height 167 cm. edema
in low extremity. Lab exams: Hb 13,8 g/dL, WBC
10.950 mL, Uric acid 7,5 mg/dL, Ureum 86 mg/dL,
Creatinin 1,4 mg/dL.
54.

What is the most appropriate nutrition for this


patient:
A.
B.
C.
D.

Restrict water to 1000 ml/day.


No restriction for food consistency.
Drink fruit juice as much as possible.
Patients should drink as much as possible.

A. He needs milk fortified calcium.


B. Restrict protein intake
C. Reduce meat base protein and
nuts.
D. Reduce protein and give high
calories.
58.

A 55 year old male complains hematuria at


thephysicians office for 2 weeks.
A cystoscopic examinations showed a 2
cm,exophytic,friable mass on the right bladder
dome.
A biopsy of this lesion show findings most
consistent with a/an
F. Papilloma
G. Verrucous carcinoma
H. Transitional carcinoma
I. Squamous carcinoma
J. Adenocarcinoma

59.

A 60 year old male has had multiple recurrent


urinary tract infections with 6 month history of
dysuria. Rectal toucher by his physician palpates
prostate enlargement.
What is the most appropiate clinical action to
diagnose carcinoma prostate ?
A. Trans rectal biopsy
B. Trans urethral biopsy
C. Transrectal FNAB
D. Rectal toucher
E. Open surgery

15

60.

A 50 year old male presents with a month long


history of flank pain and hematuria.An USG
abdominal results a 3 cm bosselated mass,
protrudes from cortical are in the right
kidney.Cytologic examination reveals malignant
cells. What is most likely biopsy finding for this
patient?
F. Wilms tumor
G. Renal Cell Carcinoma
H. Transitional carcinoma
I. Squamous carcinoma
J. Undifferentited carcinoma

61.

A 60 year old male experiences the onzet of


headache, naussea ,vomitting fo 1
month.Physical examination reveals
hypertention, subfebril. Pyelogram showed the
affected kidney assymmetrically contracted with
deformity of the calyceal system. Gross of the
affected kidney showed contracted and has
irregular granular surface.The parenchyma is
atrophic and replace by fibrosis. Microscopic
showed glomerular sclerosis, hyalinization and
atrophi in cortical area. Another areas showed
fibrosis, chronic inflammatory cells with
limpocytes agregation.
What is the most likely lesions occur in this
kidney?
F.
G.
H.
I.
J.

62.

63.

64.

A 34-year-old man comes with 3-day history of


low back pain, urinary hesitancy, and pain with
urination. He has not had any urethral discharge
or recent sexual contacts. His temperature is
38.2C, and blood pressure is 120/70 mm Hg.
Pulse 96x/min, RR 20x/min. There is no
costovertebral angle tenderness. Abdominal
examination shows no tenderness or masses.
Rectal examination shows no tender prostate
and no masses.
Specific gravity 1.030; Blood negative; Glucose
negative; Ketones negative;
Leukocyte esterase positive
What is the most likely diagnosis?
A) Benign prostatic hypertrophy
B) Cystitis
C) Prostatitis
D) Pyelonephritis
E) Urolithiasis
65. A 63 year old female patient came with complain
of pain while urinate. She has history of
recurrent urinary infections. Physical
examination revealed tenderness in CVA region.
Urinalysis was done and radiology finding
reveals large, bilateral "stag horn" calculi.
What kind of stone is likely to be demonstrated
in this case?

Renal cell carcinoma


Chronic pyelonephritis
Acute nephritis
Nephrotic syndrome
Diabetic nephropathy

A 5 year old boy child who had a palpable


abdominal mass brought to hospital by his
mother . The child later had abdominal distention
from bowel obstruction. An USG reveals a 6 cm
left renal mass with necrosis and hemorrhage.
Microscopic showed embryonal tissue with
abortive of tubular and glomerular structures.
What is the diagnosis ot this patient?
F. Teratoma
G. Retinoblastoma
H. Wilmstumor
I. Hanarthoma
J. Hemangioma

A 62-year-old woman presents with hematuria


and left flank pain. Physical examination reveals
costovertebral angle tenderness. Renal
ultrasound shows marked left hydronephrosis.
Serum creatinine and CBC are normal.
Which of the following diagnostic test is the most
relevant?
A. Renal biopsy
B. Renal arteriogram
C. Palpation for inguinal lymph nodes
D. Antinuclear antibody test
E. IVP

A. Calcium oxalate stones


B. Struvite stones
C. Calcium phosphate stones
D. Uric acid stones
E. Cystine stones
66.

A 55-year-old man is brought to the emergency


due to acute onset of severe right-sided flank
pain. He has a 10-year history of gout. His
blood pressure is 110/80 mm Hg, pulse is
78x/min, and RR 16x/min. Examination shows
normal bowel sounds and no abdominal
tenderness or masses. Urinalysis shows 40
erythrocytes/hpf.
Intravenous
pyelography
confirms a right ureteral calculus.
Which of the following underlying mechanism is
responsible for this patient's condition?
A. An increase in urinary pH
B. Damage to the epithelial lining of the
ureters
C. Lack of inhibitors of crystal formation
D. Presence of urease-splitting bacteria
E. Urinary supersaturation with uric acid

A 66 years old, came to Emergency Department with


shortness of breath as his chief complaint. He also got
nausea, vomiting and loss of appetite since 3 days
ago. No diarrhea was occurred prior to the symptoms.
He was taking un regularly anti hypertension drugs for
20 years. Physical exam: weight 70 kg, blood pressure
190/110, Pulse 120 times/minute, and Respiratory rate
46 times/minute. Pale conjunctiva, fine crackles +/+

16

were heard on the both side of the lung. Heart, liver


and spleen were normal.
Lab results: Hb 7.2 g/dl, WBC 7000/mm3, trombosit
225000/mm3, random glucose 135 mg/dl, Ureum 150
mg/dl, Creatinin 5.2 mg/dl. Proteinuria +3
67. What is the most possible causes of anemia
based on the case above?
A. B6 deficiency
B. Erithropoeitin deficiency
C. B12 deficiency
D. Iron deficiency
E. Folic deficiency
68. What is the most possible electrolyte level show?
A.
Phosphate decreased
B.
Calcium increased
C.
Potassium decreased
D.
Phosphate increased
E.
Normal Calcium
69. Mrs Jane 35 years old, came to Internal
Department with complain, fatigue and
headache about 1 month . In physical
examination: pale, butterfly rash at her face,
another examination in normal limit. The physician
suspects that Jane suffers SLE.
Whats the most important lab examination to
support the diagnosis:
a) Complete Blood Count
b) Complement fixation test
c) Anti Nuclear Antibody
d) Renal function test
e) Liver function test
70. According to case no 1, beside protein uria what is
the most urinalysis result in
this case :
a) Glucosuria
b) Leukocyte uria
c) Hematuria
d) Bilirubinuria
e) Crystaluria
71. A 25 years old man has been crashed by a truck.
The doctor suspects that his kidney is rupture, then he
sent the patient to have IVP. What is the most likely
the appearance of renal parenchymal rupture in IVP?
a. The kidney shows non function
b. Ekstravasation
c. Hydronephrosis
d. Normal nephrogram
72. What is the most likely the appearance of renal
artery rupture?
a. The kidney shows non function
b. Ekstravasation
c. Hydronephrosis
d. Normal nephrogram
73. A 50 years old man comes to the clinic with
complain of abdominal colic. The result of IVP
demonstrates ureteric dilatation.What is the most

likely the problem of ureteric dilatation?


a. Stone in the pelvicureter junction
b. Kidney cyst
c. Stone in the vesicoureter junction
d. Renal cell Ca stage I
74. A 65 years old man is diagnosed with Prostates
hypertrophy. What is the most likely feature appeared
in cysptography?
a. Indentation of inferior aspect of bladder
b. Indentation of superior aspect of bladder
c. Indentation of posterior aspect of bladder
d. Additional shadow of superior aspect of
bladder
75 A 45 years old woman is diagnosed with Bladder
stone. What is the most likely feature appeared in
cysptography?
a. Indentation of inferior aspect of bladder
b. Indentation of superior aspect of bladder
c.
Filling
defect
in
the
bladder
d. Additional shadow of superior aspect of bladder

1. 109 284. The consumption of oxygen by the


kidney
a. Decreases as blood flow increases
b. Is regulated by erythropoietin
c. Remains constant as blood flow
increases
d. Directly reflects the level of sodium
transport
e. Is greatest in the medulla
2. 110 285. The anion gap will increase with an
increase in the plasma concen-tration of
a. Sodium
b. Potassium
c. Chloride
d. Bicarbonate
e. Lactate
3. 111 288. In the presence of ADH, the filtrate will
be isotonic to plasma in the
a. Descending limb of the loop of Henle
b. Ascending limb of the loop of Henle
c. Cortical collecting tubule
d. Medullary collecting tubule
e. Renal pelvis
4. 112 289. Sodium reabsorption from the distal
tubule will be increased if there
is an increase in
a. Plasma potassium concentration
b. Plasma volume
c. Mean arterial pressure
d. Urine flow rate
e. Plasma osmolality
5. 113 290. ADH will be released from the posterior
pituitary when there is a decrease in
a.Plasma Na+concentration
b.Plasma volume
c.Plasma K+concentration
d.Plasma pH
e.Plasma Ca2+concentration

17

6. 114 291. If 600 mL of water is ingested rapidly,


plasma volume will increase by approximately
a400 mL
b200 mL
c100 mL
d50 mL
e25 mL
7. 116 293. Renin secretion by the kidney is
increased by
a. Increasing mean blood pressure
b. Increasing glomerular filtration rate
c. Increasing sympathetic nerve activity
d. Increasing angiotensin II synthesis
e. Increasing atrial natriuretic hormone
secretion
8. 117 294. Na+is reabsorbed from the basolateral
surface of the renal epithelial
a. cells by
b. a Na/H exchange
c. Na-glucose cotransport
d. Na-K pump
e. Facilitated diffusion
f. Solvent drag
9. 118 295. Which of the following is most likely to
cause an increase in the glomerular filtration
rate?
a. Contraction of mesangial cells
b. Blockage of the ureter
c. Release
of
renin
from
the
juxtaglomerular apparatus
d. Dilation of the afferent arterioles
e. Volume depletion
10. 119 296. The daily production of hydrogen ion
from CO2 is primarily buffered by
a. Extracellular bicarbonate
b. Red blood cell bicarbonate
c. Red blood cell hemoglobin
d. Plasma proteins
e. Plasma phosphate
11. 120. Glomerular filtration rate would be
decreased by
a. Constriction of the efferent arteriole
b. An increase in afferent arteriolar
pressure
c. Compression of the renal capsule
d. A decrease in the concentration of
plasma protein
e. An increase in renal blood flow
12. 121. A freely filterable substance that is
neither reabsorbed nor secreted has a renal
artery concentration of 12 mg/mL and a renal
vein concentration of 9 mg/mL. Calculate the
filtration fraction (GFR/RPF).
a. 0.05
b. 0.15
c. 0.25
d. 0.35
e. 0.45
13. 122. Use the following laboratory data to
determine the GFR.
Urine creatinine concentration =196 mg/mL
Plasma creatinine concentration=1.4 mg/mL
Urine flow =1 mL/min

The creatinine clearance is approximately


a. 75 mL/min
b. 98 mL/min
c. 125 mL/min
d. 140 mL/min
e. 196 mL/min
14. 123. The electrically neutral active transport of
sodium from the lumen of the kidney occurs in
the
a. Proximal tubule
b. Descending limb of the loop of Henle
c. Ascending limb of the loop of Henle
d. Cortical collecting duct
e. Medullary collecting duct
15. In metabolic acidosis caused by diabetic
ketoacidosis, which of the following would be
greater than normal?
a. Concentration of plasma HCO3
b. Anion gap
c. Arterial PCO2
d. Plasma pH
e. Blood volume
16. 125. Decreasing the resistance of the afferent
arteriole in the glomerulus of the kidney will
decrease
a. The renal plasma flow
b. The filtration fraction
c. The oncotic pressure of the peritubular
capillary blood
d. The glomerular filtration rate
e. None of the above
17. 126. If GFR increases, proximal tubular
reabsorption of salt and water will increase by
a process called glomerulotubular balance.
Contributions to this process include
a. An increase in peritubular capillary
hydrostatic pressure
b. A decrease in peritubular sodium
concentration
c. An increase in peritubular oncotic
pressure
d. An increase in proximal tubular flow
e. An increase in peritubular capillary
flow
18. 127. Renin release from the juxtaglomerular
apparatus is inhibited by
a. Beta-adrenergic agonists
b. Prostaglandins
c. Aldosterone
d. Stimulation of the macula densa
e. Increased pressure within the afferent
arterioles
19. 128. Patients with renal insufficiency develop
very high plasma concentrations of urea
(uremia) because of
a. An increased synthesis of urea by the
liver
b. An increased reabsorption of urea by
the proximal tubules
c. A decreased secretion of urea by the
distal tubules
d. A decreased glomerular filtration rate

18

e. An increased renal blood flow


20. 129 . Which one of the following statements
about aldosterone is correct?
a. It produces its effect by activating
cAMP
b. It produces its effect by increasing
distal tubular permeability to sodium
c. It causes an increased reabsorption of
hydrogen ion
d. It has its main effect on the proximal
tubule
e. It is secreted in response to an
increase in blood pressure
21. 130 . The effect of antidiuretic hormone (ADH)
on the kidney is to
a. Increase the permeability of the distal
nephron to water
b. Increase the glomerular filtration rate
c. Increase the excretion of Na+
d. Increase the excretion of water
e. Increase the diameter of the renal
artery
22. 131 . The glomerular filtration rate will increase
if
a. Sympathetic nerve activity to the
kidney increases
b. The afferent arteriolar resistance
increases
c. The efferent arteriolar resistance
decreases
d. The plasma protein concentration
decreases
e. Urine flow through the urethra is
blocked
23. 132 . Potassium-sparing diuretics inhibit
Na+reabsorption in the
a. Proximal tubule
b. Thin descending limb of Henles loop
c. Thick descending limb of Henles loop
d. Distal convoluted tubule
e. Cortical collecting duct
24. 133. Which one of the following values will be
above normal in a diabetic
a. patient
with
a
blood
glucose
concentration of 600 meq/L?
b. Urine flow
c. Intracellular volume
d. Plasma sodium concentration
e. Arterial pH
f. Alveolar PCO2
25. 134 . Which one of the following will be
increased in a patient suffering from persistent
diarrhea?
a. The filtered load of HCO3
b. The production of ammonia by the
proximal tubule
c. H+secretion by the distal nephron
d. The anion gap
e. The production of new bicarbonate by
the distal nephron
26. 135 . Diuretics, such as acetazolamide, which
produce their effect by
a. inhibiting carbonic anhydrase, inhibit

27.

28.

29.

30.

the reabsorption of sodium in


b. The proximal tubule
c. The thick ascending limb of Henles
loop
d. The distal convoluted tubule
e. The cortical collecting duct
237. Which is a common finding in acute
glomerulonephritis?
a. Pulmonary congestion due to volume
expansion
b. Hypovolemia
due
to
tubular
dysfunction
c. Uniformly progresses to chronic renal
failure if untreated
d. Urine
showing
leukocytes
and
eosinophils
238. Which finding is fairly specific for chronic
renal failure?
a. Anemia
b. Hyaline casts
c. Broad casts in urinalysis
d. Proteinuria
e. Hypocalcemia
239. Nephrotic syndrome is associated with
a. Excessive renal salt and water loss
b. Hyperlipidemia due to lipoprotein
excess
c. Bleeding due to loss of clotting factors
d. Hypothyroidism due to loss of thyroidbinding globulin
e. e. The outer medullary collecting duct
240. A patient with chronic renal failure will be
expected to have which of
a. the following findings due to the
mechanisms described?
b. Hypercalcemic due to elevated PTH
hormone
c. Prolonged bleeding due to decreased
synthesis of clotting factors
d. Anemia due to increased red cell
destruction
e. Hypermagnesemia due to decreased
renal excretion

31. 241. A high fractional excretion of sodium is


typically found in
a. Heart failure
b. Urinary tract obstruction
c. Acute tubular necrosis
d. Acute glomerulonephritis
e. Hepatorenal syndrome
32. 242. Which of the following nephron segment
is correctly paired with its function?
a. Distal
tubule
and
bicarbonate
reclamation
b. Loop of Henle and potassium
regulation
c. Proximal
tubule
and
urinary
concentration
d. Collecting tubule and water regulation
33. 243. Which of the following statements is true
in the management of acute renal failure?
a. Metabolic acidosis is fully corrected

19

34.

35.

36.

37.

38.

with bicarbonate
b. Hyperphosphatemia
is
primarily
managed with dialysis
c. Low-dose dopamine is used to
shorten the duration of renal failure
d. Hypervolemia is managed with highdose loop diuretics
e. Hyponatremia
is
corrected
by
administration of sodium salts
244. Which of the following describes bone
abnormalities in patients with chronic renal
failure?
a. Osteitis fibrosis cystica is a result of
oversuppression of PTH
b. Adynamic bone disease is associated
with myopathy
c. Osteomalacia is due to excessive
accumulation of magnesium
d. Hyperparathyroidism responds well to
1,25 dihydroxyvitamin D
e. Amyloidosis is similar in etiology to
patients who are not on dialysis
245. Which one of the following statements is
true concerning hematologic disorders in
CRF?
a. Resistance to erythropoietin is most
commonly due to aluminum overload
b. Erythropoietin
administration
is
associated
with
worsening
hypertension
c. The major cause of death in CRF is
sepsis
d. Abnormal bleeding responds best to
platelet transfusion
e. Leukocyte function is generally
unimpaired
246. Which of the following measures has not
been shown to retard progression of renal
failure?
a. Aggressive BP control
b. Decrease in protein intake
c. ACE
inhibitors
above
other
antihypertensives
d. Erythropoietin for anemia
247. In patients with chronic renal failure,
which of the following adaptations are normal?
a. Fractional
excretion
of
sodium
increases due to suppression of
aldosterone
b. Metabolic acidosis due to loss of
bicarbonate in the urine
c. Increased potassium loss through
extrarenal mechanisms
d. Decreased fractional excretion of
water due to ADH resistance
248. Which of the following serologic finding is
associated with linear staining of the
glomerulus on immunofluorescence?
a. Anti-GBM antibody
b. Low complement immune complex
glomerulonephritis
c. ANCA associated renal disease
d. Membranoproliferative

glomerulonephritis
39. 249. Antineutrophil cytoplasmic antibody
(ANCA) is typically present in which systemic
disease?
a. Goodpastures syndrome
b. Wegeners granulomatosis
c. Systemic lupus erythematosus
d. Thrombotic thrombocytopenic purpura
40. 252. Prerenal azotemia is associated with
a. High fractional excretion of sodium
b. Granular casts in the urine
c. Use of angiotensin-converting enzyme
(ACE) inhibitors in unilateral renal
d. artery stenosis
e. Evolution to acute tubular necrosis if
untreated
41. 254. Leukocytes and white cell casts in the
urine are typically seen in
a. Radiocontrast nephropathy
b. Methicillin-induced renal insufficiency
c. Aminoglycoside nephrotoxicity
d. Rhabdomyolysis
42. 256. Which may cause acute renal failure in
patients with nephrotic syndrome?
a. Dietary protein restriction
b. ACE inhibitors
c. Lipid-lowering agents
d. Loop diuretics
43. 264. A 25-year-old man with flank pain is
found to have three cysts in each kidney,
normal hepatic and renal function, and family
history is not clear. He is most likely to have
a. Autosomal dominant polycystic kidney
disease
b. Autosomal recessive polycystic kidney
disease
c. Acquired cystic disease
d. Medullary sponge kidney
44. 266. Which is an accurate statement
concerning diabetic nephropathy?
a. Most patients with type 2 diabetes will
develop this problem
b. It is almost always associated with
retinopathy in type 1 diabetes
c. ACE inhibition is only indicated for
patients with hypertension
d. Routine dipstick urine should be
performed to screen for early disease
45. 267. Which of the following is a secondary
cause for focal segmental sclerosis?
a. Hodgkins disease
b. Colon cancer
c. HIV disease
d. Hepatitis C infection
46. 268. A patient with Crohns disease passes a
kidney stone; the most likely
a. composition is
b. Calcium phosphate
c. Uric acid
d. Struvite
e. Calcium oxalate
47. 277. Hyperkalemia may be caused by
a. Trimethoprim

20

b. Albuterol
c. Licorice
d. Cisplatin
48. 278. A middle-aged patient with an elevated
serum creatinine, hypertension, and mild
anemia comes to you for evaluation. Urine
dipstick shows trace protein without red cells
or cellular casts. A 24-h urine collection
reveals 5 g of protein. The most likely etiology
is
a. Focal segmental sclerosis
b. Hypertensive nephrosclerosis
c. Amyloidosis
d. Multiple myeloma
49. 279. Which of the following is a common
cause of isolated hematuria with isomorphic
red cells in the urine?
a. Alports
syndrome
(hereditary
nephritis)
b. Thin basement membrane disease
c. Idiopathic hypercalciuria
d. IgA nephropathy
50. 280. A 26-year-old woman with a history of
mitral valve prolapse comes in with 1 week of
fever that started 3 days after a dental
procedure. Her urine contains red cells and
her rheumatoid factor is elevated. Which of the
following serologic abnormalities is expected
to be present?
a. Anti-GBM antibody
b. Low serum complement levels
c. Antineutrophil cytoplasmic antibody
d. Elevated IgA levels
51. 440. A 28-year-old previously healthy female,
with no medical history is now 28 weeks
pregnant. She complains of trouble seeing,
polyuria, polyphagia, and polydipsia. What is
her diagnosis?
a. Gestational diabetes mellitus
b. Deep venous thrombosis
c. Urinary tract infection
d. Preeclampsia
52. 4-38. A 48-year-old man presents with
peripheral edema. He has been healthy and
physically active all of his life. His family
history is unremarkable. His blood pressure is
normal. On physical examination, the patient is
noted to have anasarca. Kidneys are not
palpable. Urinalysis reveals a moderate
amount of proteinuria and grape clusters are
seen under light microscopy. Which of the
following is the most likely diagnosis?
a. Glomerulonephritis
b. Rhabdomyolysis
c. Nephrotic syndrome
d. Acute interstitial nephritis
e. Acute tubular necrosis
53. 6-31. A 50-year-old woman com- plains of
leakage of urine when she laughs, coughs, or
sneezes. After stress incontinence, the most
common causes of this urinary leakage is

a.
b.
c.
d.
e.

Detrusor dyssynergia
Unstable bladder
Unstable urethra
Urethral diverticulum
Overflow incontinence

1. To elevate the kidney within the renal fascia


and the perirenal fat, the renal fascia must be
reflected or incised from the fascia of the?
A.
B.
C.
D.
E.
2.

diaphragm
psoas m.
quadratus lumborum m.
transverse abdominis m.
iliacus m.
Young man, vomit to point where he become
hypovolemic; as evidence by an
accompanying decrease in BP and feeling of
light-headness. The kidney respond by
reducing urinary volume flow, thus limiting the
potential effect of hypovolemia. increase in the
plasma level of which of the following
hormones will bring about the most dramatic
decrease in urinary volume flow rate?

A.
B.
C.
D.
E.
3.

angiotensin II
atrial natriuretic peptide
PTH
aldosterone
ADH
56 y.o. woman is diagnosed w/ small cell lung
ca. she has paraneoplastic effect from the ca,
which release of an atidiruetic hormone-like
agent. which of the following is the most likely
to be seen?

A.
B.
C.
D.

elevated serum sodium


elevated serum osmolarity
elevated urine sodium
elevated urine cathecolamines

4. Histological sections from an abdominal mass


that was removed from a 13 month old female
reveal undifferentiated mesenchymal cell,
immature tubules, and abortive glomerular
formation. What is the best diagnosis for this
tumor?
A. Dupuytren
B. Ewing
C. Ollier
D. Warthin
E. Wilms
5. A 63 y.o. woman has type II DM. PE positive
for peripheral neuropathy in the feet and
nonproliferative retinopathy. Urinalysis positive
for proteinuria. which of the following treatment
is most likely to attenuate the course of renal
disease?
A. calcium channel blocker
B. ACE inhibitor
C. HMG-CoA inhibitor

21

D. dietary carbohydrate restriction


E. weight reduction
6. Ingesting antacids with and after a meal so
that gastric pH does not decrease below pH 6
will cause a greater than normal secretion of?
A. gastrin
B. secretin
C. pancreatic bicarbonate
D. cholecystokinin
E. somatostatin
7. A 16 y.o. girl is reffered to the office because
of chronic diarrhea and weight loss. She is
experiencing large volume watery diarrhea
that is painless. The symptoms persist even
when shes fasting, and there is no
relationship to foods or liquids. Shes not on
any medication, and there is no travel history
or other constitutional symptoms. Her PE is
normal. Which of the following is the most
likely diagnosis?
A. partial small bowel obstruction
B. partial large bowel obstruction
C. osmotic diarrhea
D. secretory diarrhea
E. IBS
8. A 42 y.o. overweight but otherwise healthy
women present with sudden onset of right
upper abdominal colicky pain in 45 minutes
after meal of ayam goreng. Pain is associated
with nausea and vomiting, and any attempt to
eat since has caused increased pain. What is
the most likely cause?
A. gastric ulcer
B. cholelithiasis
C. duodenal ulcer
D. acute hepatitis
E. esophageal spasm
9. A 42 y.o. male, executive, complains of
abdominal pain that began about 6 months
ago, is constant in nature especially after
meals, and located in the upper midabdomen
superior to umbilicus. He also report some
heartburn that has been occurring during the
previous year. He as been under a lot of jobrelated stress. His stool have changed in color
over the previous 2 months and now are
intermittently dark and tarry in consistency.
The physician test the patient stool and find
occult fecal blood. What is the most likely
diagnosis?
A. peptic ulcer
B. gastritis
C. Chrons disease
D. gastric polyp
10. A 38 y.o banker with a history of heartburn
suddenly experieces excruciating pain in the
epigastric region of the abdomen. Surgery is
performed immediately on admission to ER.

A.
B.
C.
D.
E.

There is evidence of an ulcer, which has


ruptured through the posterior wall of
duodenum. What blood vessel might be
subject to erosion?
common hepatic
left gastric
splenic
superior mesenteric
gastroduodenal
KIDNEY

54. 42 year old man, muscular weakness, mild


headache after eating salted fish. Small
amount water drink. Usual BP 100/70, Now
become 140/90, Na : 150 mEq/ml, K : 3,5,
glucose 150. efek?
a. Increase GFR
b. Increase obligatory water reabsorbtion
c. Increase facultative water
reabsorbtion
d. Increase secretion
e. Increase aldotesteron secretion
55. To cover above function (no 1), what
mechanism
a. Increase cardiac output
b. Increase frekuensi heart
c. Increase secretion rennin
d. Increase secretion ADH
e. Increase secretion aldosteron
56. Factor yang mensupport prosess diatas ialah
a. Filtration late of glomeruli
b. Active transport of tubular cell
c. High osmolalitas of interstitium of
medulla
d. Obligatory water reabsorbtion
e. Osmalalitas of filtrate in distal
contortus tubulus
57. Pasien dengan heart failure = low CO, perfusi
hanya organ-organ penting. Obat yang tidak
bole diberikan? thiazide
58. Pasien dengan heart failure, diberi obat
Cloro..tiazid/mid dengan maximum dose tidak
merespon. Diberi apa?furosemid
59. Pasien diberikan riazid selama tahun. Efek
yang terjadi ! decrease kalium
60. Mountain sickness merupakan side effect?
dari?
a. czetazolamide->bukan side effect, tp
profilaksis
b. Budenamide
c. Amyloride
d. Furosemide
e. Spironolactone
61. Seorang pasien sedang menjalani
pengobatan, pasien complaint Bright orangered urine merupakan side effect dari obat?
a. Ethambutol
b. Isoniazid
c. Piridizone
d. Rifampisin
e. Streptomisin

22

62. Pasien mengeluh dengan complaint(symtomp


gonorrhea) treatmentnya?
a. Apnofloxacin
b. Nalydixic acid
c. Norifloxacin
d. LEvofloxacin-e. Ofloxacin
63. Jaundice 1 yo, increase blirubinn. Ibunya
diberi antibiotic for UTI, baby mengalami
kernicterus ,disebabkan karena obat !
cotrimoxazole
64. 30 yo waman recurrent UTI, culture E coli di
kase aprofloxacin. Penghambat apa?Dna
gyrase
65. 30 yo man motor accident most anterior within
renal sinus. ! Renal vein
66. You wish examine the hilum of the left kidney
during surgery. Which pf following structure
must be elevated. pancreas
a. Stomach
b. Suprarenal gland
c. Ascending colon
d. Duodenum
e. Liver
67. To elevate kidney within the renal fascia dan
perirenal fat which of following muscle
a. Diafragma
b. Psoas
c. Quadratus lumborum
d. Transverse abdominis
e. Illeus
68. Laki-laki probable testicular cancer, which
layer is the most near to testicular parenchyma
a. FS xterna
b. Tunica albuginea
c. FS interna
d. Scarpa interna
e. Tunica dartos
69. 18 tahun laki-laki. Dysuria for several day
UTI. Structure yang vulnerable get infection?
a. Kidney
b. Ureter
c. Bladder
d. Uretra
e. Adrenal gland
70. 70 laki-laki came to emergency room.
Incomplete urination since 6 bulan lalu.
Structure yang change?
a. Uretral pars membranosa
b. Uretral pars spongiosa
c. Uretral pars prostatica
d. Uretral orificium uretra
71. During 4 weeks treatment, the urine protein +1
, so the diagnosis :
a. Initial responder
b. Lat responder
c. Drug resistant
d. Drug dependent
e. Remission
72. 13 years old boy comes to ER with seizure,
fever 2 weeks but has recovered, swelling in
the eyes and legs, dark urine. Unconscious ,

bp 180/120. Urine erythrocyte 20/hpf, protein


+4, albumin normal. Lab finding
a. ASO decrease, dnase-b increase, c3
decrease
b. ASO increase, dnase-b decrease, c3
decrease
c. ASO decrease, dnase-b increase, c3
increase
d. ASO increase, dnase-b decrease, c3
decrease
e. ASO increase, anti dnase-b increase,
c3 decrease
Anti-DNase B is a blood test to look for a substance
produced by Group A Streptococcus, the bacteria that
cause strep throat.
73. Underlying mechanism dari pertanyaan 19 ?
a. Streptococcal infection
b. Inflammation of kidney tubule
c. Increase permeability of glomerulus
membrane
d. Small pores in podocyte permit rbc
pass through
e. Heparansulfat polysaccharide is low,
so rbc could cross the BBB
74. Anak di rawat karena indirect bilirubinemia. 3
hari kemudian demam, dan diberi gentamicin
dan amoxicilin. Kenaikan BUN dan creatinine.
Apa mechanisme nya?
a. ATN
b. AIN
c. AKI
d. CKI
75. Kasus no 21 bisa terjadi karena Disebabkan
oleh?
a. Iskemik
b. Toxic injury
c. Drug hypersensitivity
d. Immune Complex
76. 10 tahun, dysuria 5 hari yang lalu. No
significcant finding, urine culture E coli >
100.000
a. Acute pyelonefritis
b. Reflux nephropathy
c. Uncomplicated UTI
d. Complicated UTI
e. Chronic UTI
77. Which of the following factor unfavorable to
bacterial growth in patients urine
a. A low pH (5,5 or <)
b. Low concentration urea
c. Abscense of organic a diet
d. Low osmolality
e. Presence of normal microorganism
78. Antibiotik untuk UTI first line
a. Ceftriaxone
b. Cefotaxime
c. Ceftrazidime
d. Cotrimoxazole
e. Metronidazole
79. Gfr <15%
Rifle criteria
a. Risk

23

80.
81.
82.
83.
84.
85.
86.
87.
88.
89.
90.

91.

92.

93.

94.

b. Injury
c. Fail
d. Loss
e. ESKD
Pada orang yang mengalami Hematuria in
bladder cancer,bagaimana bentuk rasa
sakitnya ! Painless and intermitten
PH urine = 5. Uric acid = 8,9
a. Lot of fluid + allopurinol
b. Lots of fluid + aciding agent
40 tahun old male. Thoracic gunwound,
hematuria. Diagnose?renal?
Bladder injury. Mostly caused by?trauma
Polyuria, kasih obat apa?vasopressin
Pneumocyctis carinii.. WBC increase etc., !
post infection glomerulonefritis
Pengaruh parasimpatis terhadap bladder !
contraction of detrusor muscle
Parasimpatic ! S2,3,4 dan cranial 3,7,9,10
Dementia ,cant holding a pee.! frontal lobe
Diarrhea 8 hari what the best treatment! RL
infusion
44 tahun laki-laki. Systemic fungi infection on
amphotericin B treatment for 7 days.
Conscious, BP 120/80, pulse 80x/min, RR
16x. 36,7 c, HB 13, HT 37, WBC 8600.
Platelet 340.000, Urinalisis RBC (0-1), WBC
(2-5), protein -, Granular cast +, Ureum 91,
creatinin 2,0, urine 800/12 jam
a. ATN
b. Acute glomerulonefritis
c. Acute interstitial nefritis
d. ARF
e. CRF
Woman, pain during urination and suprapubic
pain, no fever, cloudy urine. Microscopic
hematuria, pyuria, granular +, no past history
a. Klebsiella pneumonia
b. P. aeruginosa
c. E coli
d. Proteus mirabilis
e. S. saphroliticus
Which urine specimen most appropriate?
Untuk culture
a. Suprapubic
b. Clean catch
c. Strecth catheter
d. Indualing catheter
e. 24 hour urine
UTI, beta hemolytic virulence factor
a. Capsule
b. P fimbrae
c. Endotoxin
d. Fosfolipase c
e. Lipopolisaccaride
UTI, - gram, coccobacillus, high pH.virulence
vactornya?
a. Hemolisin
b. Fibronecrine
c. Catalase
d. Cordially
e. Opaprotein

95. Pada orang yang pernah mengalami atau


sedang mengalami kidney stone,tindakan
preventive apa yang diperlukan agar tidak
terjadi penambahan atau recurrenci dari batu
ginjal tersebut?
a. Restriksi air 100 ml
b. Minum jus yang banyak
c. Minum sebanyak-banyaknya
d. Minum pas haus
e. Makan buah
96. 60 tahun, headache, nausea, vomit 1 bulan.
PE : HT, subfebrile. Pyelogram showed
affected kidney asimetri contracted with
deform of calyceal system. Gross of the
affected kidney showed contacted dan has
irregular granular surface. Parenkim atropi and
replace fibrosis. Microscopi : glomerular
sclerosis, hyalinisasi, atropi incortical. Another
area : fibrosis, chronic inflammation cell with
limfosit aggregation
a. RCC
b. Pyelonefrits
c. Nefritis
d. NS
e. Diabetic nefropathy
97. 5 years old, pale, abdominal mass. Distention
to bowel e=obstraction. USG : 6 cm left renal
mass with necrosis and hemorrhage. Micros :
embrional tissue with abbortive tubular and
glomerular structure ! Wilms tumor
98. 62 tahun, hematuri, left flank pain. PE :
costovetebral angle tender. USG : left
hydronefrosis. CN and CBC .
a. Renal biopsy
b. R. ateriogram
c. Palpasi lymph node
d. ANA
e. IVP
99. Renal trauma, terlindas truk, ginjal kena
kerusakan. Parenkimnya rusak. Maka yang
terjadi pada ginjal tersebut ialah?
a. Ginjal tidak berfungsi
b. Ekstravasasi
c. Hidronefrosis
d. Normal urogram
100.
Injury kena renal artery
101.
Pada pasien dengan pemeriksaan
Radiology menunjukkan ada pelebaran ureter,
maka diperirakan batu berada di?
a. Vesikoureter
b. Pelvicc ureter
102.
65, laki-laki, Ada bladder dengan
perbesaran prostat, kama radiografi. Gimana
!
a. penonjolan dari inferior bladder
b. Penonjolan dari superior bladder
c. Penonjolan dari superior bladder
d. Penonjolan dari posterior bladder
103.
50 tahun colic. IVP = uteric dilatation.
Kenapa bisa dilatasi?
a. Stone in pelvicureter junction
b. Kedney cysts

24

c. Stone in the vesicoureter junction


d. Renal cell CA stage 1
104.
45 wanita, bladder a stone.
Cystography
a. Indertaion of inferior aspect of bladder
b. Indertation of superior aspect of
bladder
c. Filling defect in the bladder
1. 4yrs boy. Puffy eye in the morning,swelling
leg, foamy urine, protein +4, serum albumin
1,5. Principle management!prednison 60mg.
2. Urine sample dari +4 ke +1 selama 1 bulan.
Patient responsive classification: initial
responder
3. Boy, severe diarrhea. Not full alert. Bp 80/60,
pulse not palpable, RR 30, temperature
38,4C, sunken fontanele etc (moderatesevere dehydration). Distal tubular susceptible
to ischemic injury : loss of polarity re
distribution of membrane protein
1. During the removal of a patients kidney, you
would observe which of the following as being
most anterior within the renal sinus?
a. Renal arteries
b. Renal vein
c. Major calyx
d. Minor calyx
e. Renal pelvis
2. You wish to examine the hilum of the right
kidney during surgery. Which of the following
structures must be elevated and reflected to
do so?
a. Stomach
b. Suprarenal gland
c. Ascending colon
d. Duodenum
e. Liver
3. To elevate the kidney within the renal fascia
and the perirenal fat, the renal fascia must be
reflected or incised from the fascia of the:
a. Diaphragm
b. Psoas muscle
c. Quadratus lumborum muscle
d. Transverse abdominis muscle
e. Iliacus muscle
4. Which of the following is the male homologue
of the female clitoris?
a. Epididimis
b. Vas deferens
c. Penis
d. Scrotum
5. The scrotum appears to have a slightly
pigmented and wrinkled appearance. What is
the explanation for this appearance?
a. Hyperkeratinized squamous epithelium
b. The tunica albugenia
c. The dartos fascia

d. The pampiniform plexus


6. An 18-years-old man is noted to have
probable testicular cancer. He undergoes
surgery. After incising the scrotum, the
surgeon contemplates the approach to the
parenchyma of the testes. Through which
layer must the surgeon incise to reach the
testicular parenchyma?
a. Bucks fascia
b. Tunica albuginea
c. Dartos fascia
d. Scarpas fascia
7. A 66-year-old man complains of difficulty
voiding and is noted to have probable BPH.
Which of the following prostatic lobes is likely
to be responsible for these symptoms?
a. Anterior lobe
b. Posterior lobe
c. Lateral lobe
d. Middle lobe
8. A 48-year-old man is undergoing cystoscopic
examination. As the cystoscope is placed into
the urethra through the penile portion, which of
the following tissue surrounds the urethra?
a. Prostate
b. Corpus spongiosum
c. Seminal colliculus
d. Spinchter urethrae mucles
9. A police detective takes a scraping of some
stains to be examined for alkaline
phosphatase to assess whether these might
be ejaculate. What is the source of alkaline
phosphatase in the semen?
a. Prostatic gland
b. Bulbourethral glands
c. Seminal vesicles
d. Seminal colliculus apparatus
10. A 39 yo woman complains of hematuria and
significant flank tenderness. She has a history
of kidney stones. A CT scan depicts the
abdominal portion of the ureter lying anterior to
a muscle. Which of the following is most likely
to be the name of this muscle?
a. Psoas
b. Serratus anterior muscle
c. Obturator muscle
d. Rectus muscle
e. External oblique muscle
11. An individual is known to be suffering from
DM. Recently, he has developed hypertension.
His doctor suspects that the patient may be
developing renal insufficiency that is leading to
a reduced glomerular filtration and, as a result,
hypervolemia and hypertension. The doctor
wishes to evaluate kidney function by
measuring the GFR. Which is the best

25

substances for estimating the GFR from the


urine?
a. Creatinine
b. Para-aminohippuric acid (PAH)
c. Urea
d. Glucose
e. Sodium
12. A 21 yo man has been vomiting to the point
where he has become hypovolemic, as
evidence by an accompanying decrease in
blood pressure and a feeling of lightheadedness. The kidneys respond by reducing
urinary volume flow, thus limiting the potential
extent of hypovolemia. Increase in the plasma
levels of which of the following hormones will
bring about the most dramatic decrease in
urinary volume flow rate?
a. Angiotensin II
b. Atrial natriuretic peptide
c. PTH
d. Aldosterone
e. ADH
13. A 56 yo woman is diagnosed with small cell
carcinoma of the lung. She has a
paraneoplastic effect from the cancer, with
release of an antidiuretic hormone-like agent.
Which of the following is the most likely to be
seen?
a. Elevated serum sodium
b. Elevated serum osmolarity
c. Elevated urine sodium
d. Elevated urine catecholamines
14. An individual has adult-onset diabetes. She
has high levels of glucose in the urine and is
experiencing a brisk diuresis. The appearance
of glucose in the urine is a consequence of
which of the following processes in the
proximal tubules?
a. Inhibition of Na-K ATPase (Na+ pump)
b. Saturation of the Na-glucose cotransporter
c. Saturation of the Na-H exchanger
d. Stimulation of glucose secretion
e. Stimulation of glycogen breakdown
15. A student under stress has been feeling lightheaded, especially after standing, and has
developed a brisk diuresis. He has the smell of
acetone on his breath. Upon admission to the
emergency room he is diagnosed with diabetic
ketoacidosis, which is accompanied by
extreme hypovolemia, supposedly because of
the brisk diuresis. The brisk diuresis is a
consequence of which of the following?
a. High levels of glucose in the tubular
fluid/urine
b. Increased GFR
c. Suppression of ADH secretion
d. Suppression of aldosterone secretion
e. Decreased angiotensin II plasma levels.

16. A hypertensive patient was placed on the


diuretic Lasix (furosemid) to increase urinary
output. Furosemid, a high-ceiling diuretic, is
a potent diuretic because it binds to and
inhibits which of the following transport
processes?
a. The Na-glucose co transporter in the
proximal tubule
b. The Na-K exchange pump in all nephron
segments
c. The Na-K-Cl cotransporter in the thick
ascending limb
d. The Na-Cl cotransporter in the distal
convoluted tubule
e. The Na channel in the cortical collecting
duct.
17. A hypertensive patient is prescribed a loop
diuretic such as Lasix without any
supplements. One week later the patient
returns to the clinic complaining of dizziness,
weakness, and nausea. The most likely cause
of the patients worsening condition is the
development of :
a. Metabolic acidosis
b. Hyponatremia
c. Hypocalcaemia
d. Hypokalemia
e. Hypovolemia
18. A 35 yo female is noted to have new onset
hypertension that is thought to be due to an
aldosterone-secreting adrenal tumor. Which of
the following is likely to be seen in this patient?
a. Hypertension markedly improved with
furosemid
b. Elevated serum sodium level
c. Elevated serum potassium level
d. Elevated urinary cortisol level
19. A renal biopsy from an adult who presented
with progressive renal failure and hematuria
reveals linear deposits of IgG within the
glomeruli. What type of autoantibody is most
likely to be present in this individual?
a. Anti-basement membrane antibodies
b. Anti-centromere antibodies
c. Anti-double-stranded DNA antibodies
d. Antimitochondria antibodies
e. Anti-smooth muscle antibodies
20. Which one of the following combinations of
signs and symptoms is most consistent with a
diagnosis of nephritic syndrome?
a. Hematuria, hypertension, and proteinuria
b. Massive
proteinuria,
edema,
and
hyperlipidemia
c. Oliguria, hydronephrosis, and abdominal
rebound tenderness
d. Painful hematuria, flank pain, and palpable
abdominal mass
e. Painless hematuria, polycythemia, and
increased skin pigmentation

26

21. A 30 yo female patient presents with a new


onset of peripheral edema. PE finds
hypertension & bilateral pedal edema.
Urinalysis finds massive proteinuria, and
evaluation of her serum finds elevated levels
of cholesterol. A silver stain of a renal biopsy
specimen reveals a characteristic spike and
dome pattern, and electron microscopy finds a
uniform deposition of small electron dense
deposits in a subepithelial location. Which one
of the following immunofluorescence patterns
is most characteristic of this patients renal
disease?
a. Granular pattern of IgA and C3
b. Granular pattern of IgG and C3-acute
glomerulonephritis
c. Linear pattern of IgD and C4
d. Linear pattern of IgE and C4
e. Linear pattern of IgM and C3
22. A 54 yo man presents with worsening pain on
the left side accompanied by gross hematuria.
Workup finds a 4.5 cm pas in the upper pole of
the left kidney. What is the characteristic
histological appearance of the conventional
type of malignancy in this location?
a. Disorganized groups of immature tubules
b. Sheets of transitional epithelial cells
c. Small cells forming numerous papillary
structures
d. Undifferentiated
cells
demonstrating
abortive glomerular formation
e. Uniform cells with clear cytoplasm
resulting from glycogen
23. Histological sections from an abdominal mass
that was removed from a 13-month-old female
reveal undifferentiated mesenchymal cells,
immature tubules, and abortive glomerular
formation. What is the best diagnosis for this
tumor?
a. Dupuytren tumor
b. Ewing tumor
c. Ollier tumor
d. Warthin tumor
e. Wilms tumor
24. A 69 yo man has lost a friend to prostate
cancer, and would like to be evaluated for the
disease. He has no urinary symptoms. Which
of the following test is most likely indicated to
screen him for prostate cancer?
a. Prostate ultrasound
b. Digital Rectal Examination (DRE)
c. DRE and prostate specific antigen (PSA)
d. PSA
e. None of above
25. A 63 yo woman has Type II DM, which is wellcontrolled. Her physical examination is positive
for peripheral neuropathy in the feet and
nonproliferative retinopathy. A urinalysis is

positive for proteinuria. Which of the following


treatment is most likely to attenuate the course
of renal disease?
a. Calcium channel blockers
b. ACE inhibitors
c. Hepatic hydroxymethylglutaryl coenzyme
A (HMG-CoA) inhibitors
d. Dietary carbohydrate restriction
e. Weight reduction
26. A 22 yo woman come to the clinic with sign of
septic shock. The doctor prescribe her to
consume antibiotic for 2 weeks. After one
week taking the medications she come to the
doctor with complain of decrease in urine
volume, and edema. On physical examination
found that the blood pressure is 140/90, heart
rate 80, and RR=20. The urinalysis show
proteinuria and muddy brown cast were found.
What is the most possible antibiotic that can
cause this symptoms?
a. Amoxycillin
b. ceftriaxone
c. cefixim
d. Levofloxacin
e. Gentamycin
27. A 69 yo woman present with left flank pain and
hematuria. Physical examination suggest a left
side abdominal mass. CT scan of abdomen
reveal 5cm mass in the left kidney. Which of
the following laboratory abnormalities might
also be present?
a. Polycythemia
b. Thrombocytopenia
c. Hypocalcemia
d. Leukocytosis
e. High rennin hypertension
28. A 60 yo man with heart failure and normal
renal function is start of furosemid (Lasix)
80mg/day. She notice a good diuretic
response every time she take the medication.
A few week later, she is feeling unwell
because of muscle weakness and fatigue, but
her heart failure are better. Which of the
following is the most likely explanation for her
muscle weakness?
a. Hyponatremia
b. Hypernatremia
c. Hypokalemia
d. Hyperkalemia
e. Anemia
Z
29. A 78 yo man is brought to the hospital
because of nausea and vomiting. On the
physical examination he appear dry, his
abdominal is soft, the JVP is not visible. His
laboratory test reveal hypernatremia and free
water deficit is approximately 3L. In what part
of the normal kidney is most of the water
reabsorb from?
a. Colleting duct
b. Proximal tubulus

27

c. Distal tubulus
d. Ascending loop of Henle
e. Descending loop of Henle
30. A 25 yo woman present with pain on his back
and abdomen. She also complain of having
fever, nausea and vomiting. About 1 year ago
she has history of renal stone and sometimes
she found some solid material in her urine. On
histopathology examination found
glomerulosclerosis, thickening of the artery,
granulation tissue, and limfosit. What is your
working diagnosis according to the
histopathology?
a. Nephroblastoma
b. Chronic pyelonephritis
c. Acute tubular necrosis
d. Cystitis
e. Acute pyelonephritis
31. A 63 yo man with an 8 year history of recurrent
severe arthritis in his large toes has dysuria.
He also had pain that comes from his back
that is radiating to the scrotum. Which of the
following mechanism is the most likely
explanation for his symptoms?
a. Uric acid kidney stones
b. ARF
c. Renal parenchymal uric acid crystals
d. Chronic pyelonephritis
e. Acute pyelonephritis

28

You might also like